Вы находитесь на странице: 1из 280

Design of Rectangular water tank CASE-1 ( L / B < 2 )

Capacity 80000 Litres


(given)
Material
M20 Grade Concrete
(given)
Fe 415 Grade HYSD reinforcement
(given)
Solution :Provide 6 m x 4 m x 3.5 m tank with free board of 150 mm.
Volume = 6 x 4 x 3.35 x 10 3 =
80400 Litres
6
/
4
=
1.5
<2.
L/B=
The top portion of side walls will be designed as a continuous frame.
bottom 1 m or H / 4 whichever is more is designed as cantilever.
H/4=
3.5 / 4 =
0.875 m
bottom 1 m will be designed as cantilever.
2.5 x YW = 2.5 x 9.8
Water pressure at 3.5 - 1 = 2.5 m height from top =
=

24.5

KN / m2

where Yw is unit weight of water = 9.8 KN / m3


To find moment in side walls, moment distribution or kani's method is used. As the
frame is symmetrical about both the axes, only one joint is solved
6m

2.5 m

3.5 m

6m
E

24.5 KN / m

1m
D
34.3 KN / m

Elevation

Plan

Fixed end moments :MAB = w x l2 / 12 =

MAD =

= 24.5 x 62 / 12 =

73.5

KNm

w x l2 / 12 =
= 24.5 x 42 / 12 =

-32.66

KNm

Kani's Method :-

0
0

- 32.66
-12.25
-12.25
0
-57.16

73.5
-3/10 40.84 -2/10 -8.17
-8.17
A
0
57.16

0
0

Rotation factor at Joint A


Joint

Member

Relative
Stiffness( K )

AB
AD

I/6
I/4

5 * I / 12

Sum of FEM
MAF = 73.5-32.66
40.84

Rotation Factor
u =(-1/2) k / K

- 2 / 10
- 3 / 10

KNm

MAB =

MABF + 2 MAB' + MBA'


= 73.5 + 2 x (- 8.17 ) + 0
= 57.16

MAD =

MADF + 2 MAD' + MDA'


= (- 32.66 ) + 2 x (- 12.25 ) + 0
= -57.16

B.M. at centre of long span =

w x l 2 / 8 - 57.16
= 24.5 x 62 / 8 - 57.16
53.09
KNm

B.M. at centre of short span =

w x l 2 / 8 - 57.16
= 24.5 x 42 / 8 - 57.16
-8.16
KNm

Yw ( H - h ) x B / 2
24.5 x 4 / 2 =

49

KN

Yw ( H - h ) x L / 2
24.5 x 6 / 2 =

73.5

KN

Direct tension in long wall =

Direct tension in short wall =

Design of Long Walls :At support


M=
57.16
T=
From Table 9-6
D=

KNm

49
KN
Tension on liquid face.
Q = 0.306 Assuming d / D = 0.9

M / Q x b
= 57.16 x 10
=

Take D = 450 mm

432.2

/ 0.306 x 1000
mm,
Assuming d / D = 0.9

d = 450 - 25 - 8

417 mm

From Table 9-5


Ast1 for moment = M / st x j x d
=
=

57.16 x 10 6 / 150 x 0.872 x 417


1048
mm2

Ast2 for direct tension = T / st


=
=
Total Ast1 + Ast2 =

49 x 10 3 / 150
327
mm2

1048 + 327
= 1375
mm2

Provide 16 mm O bar
spacing of bar = Area of one bar x 1000 / required area in m 2 / m
= 200.96 x 1000 / 1375
= 146.152727
mm
Provide 16 mm O bar @ 130 mm C/Cmarked(a)
= 1546
mm2 / m.
Larger steel area is provided to match with the steel of short walls.
At centre
M=
53.09
KNm
T=
49
KN
tension on remote face
e=
M/T=
53.09 / 49
= 1.08 m
Line of action of forces lies outside the section
i.e.tension is small
E=
e+D/2-d
b
= 1080 + 450 / 2 - 417
=
888
mm
D

modified moment = 49 x 0.888


=
43.51

KNm
d'

Ast1 for moment = M / st x j x d


=
=

E=e+D/2-d

43.51 x 10 6 / 190 x 0.89 x 417


617

mm2

Ast2 for direct tension = T / st


=
=
Total Ast1 + Ast2 =
Provide 16 mm O bar

49 x 10 3 / 150
327
mm2

617 + 327
= 944

mm2

spacing of bar =

Area of one bar x 1000 / required area in m 2 / m


= 200.96 x 1000 / 944
= 212.881356
mm
Provide 16 mm O bar @ 200 mm C/Cmarked(b)
= 1005
mm2
From Table 9-3
minimum reinforcement 0.16 %
Distribution steel = 0.16 / 100 x 1000 x 450
=
720
mm2
On each face =
360
mm2
Provide 8 mm O bar
spacing of bar = Area of one bar x 1000 / required area in m 2 / m
= 50.24 x 1000 /360
= 139.555556
mm
Provide 8 mm O bar @ 130 mm C/Cmarked(d)
= 385
mm2
Vertical Steel ( c)
Provide 10 mm O bar
spacing of bar = Area of one bar x 1000 / required area in m 2 / m
= 78.50 x 1000 /360
= 218.055556
mm
Provide 10 mm O bar @ 200 mm C/Cmarked(c)
= 392
mm2
Horizontal steel :Remote face ( b) - Provide 16 mm O @ 200 mm C/C = 1005
mm2
Liquid face ( d) - Provide 8 mm O @ 130 mm C/C =
385
mm2
Vertical Steel ( c) - Provide 10 mm O @ 200 mm C/C both faces = 392
Design of short walls :At support
M=
T=
From Table 9-5
Ast1 for moment = M / st x j x d
=
=

57.16
73.5

KNm
KN

tension on liquid face

57.16 x 10 6 / 150 x 0.872 x 417


1048
mm2

Ast2 for direct tension = T / st


=
=
Total Ast1 + Ast2 =

73.5 x 10 3 / 150
490
mm2

1048 + 490
= 1538
mm2

Provide 16 mm O bar
spacing of bar = Area of one bar x 1000 / required area in m 2 / m
= 200.96 x 1000 / 1538
= 130.663199
mm
Provide 16 mm O bar @ 130 mm C/Cmarked(b)
= 1546
mm2 / m.

mm2

At centre
M=
8.16
T=
73.5
From Table 9-5
Ast1 for moment = M / st x j x d
=
=

KNm
KN

tension on liquid face

8.16 x 10 6 / 150 x 0.872 x 417


150
mm2

Ast2 for direct tension = T / st


=
=
Total Ast1 + Ast2 =

73.5 x 10 3 / 150
490
mm2

150 + 490
= 640

mm2

Provide 12 mm O bar
spacing of bar =

Area of one bar x 1000 / required area in m 2 / m


= 113.04 x 1000 / 640
= 176.625
mm
Provide 12 mm O bar @ 130 mm C/Cmarked(e)
= 869
mm2 / m.
From Table 9-3
minimum reinforcement 0.16 %
Distribution steel = 0.16 / 100 x 1000 x 450
=
720
mm2
On each face =
360
mm2
Provide 8 mm O bar
spacing of bar = Area of one bar x 1000 / required area in m 2 / m
= 50.24 x 1000 /360
= 139.555556
mm
Provide 8 mm O bar @ 130 mm C/Cmarked(d)
= 385
mm2
Vertical Steel ( c)
Provide 10 mm O bar
spacing of bar = Area of one bar x 1000 / required area in m 2 / m
= 78.50 x 1000 /360
= 218.055556
mm
Provide 10 mm O bar @ 200 mm C/Cmarked(c)
= 392
mm2 .
Horizontal steel :Remote face ( e) - Provide 12 mm O @ 130 mm C/C = 869
mm2
Liquid face ( d) - Provide 8 mm O @ 130 mm C/C =
385
mm2
Vertical Steel ( c) - Provide 10 mm O @ 200 mm C/C both faces = 392
Bottom 1 m will be designed as cantilever

mm2

Cantilever moment : Yw x H x h2 / 6
M=
= 9.8 x 3.5 x 1 / 6
= 5.72
KNm
From Table 9-5
Ast for moment =

OR

Yw x H / 6 , whichever is greater.
= 9.8 x 3.5 / 6
=
5.72
,tension on liquid face.

M / st x j x d
=
=

5.72 x 10 6 / 150 x 0.872 x 417


105
mm2
From Table 9-3
minimum reinforcement 0.16 %
Distribution steel = 0.16 / 100 x 1000 x 450
=
720
mm2
On each face =
360
mm2
Provide 10 mm O bar
spacing of bar = Area of one bar x 1000 / required area in m 2 /m
= 78.50 x 1000 /360
=
218
mm
Provide 10 mm O bar @ 200 mm C/Cmarked(c)
= 392
mm2 .
each face
Base slab :Base slab is resting on ground. For a water head 3.5 m, provide 150 mm thick slab.
From table 9-3
Minimum steel =
0.229%
= 0.229 / 100 x 1000 x 150
=
344
mm2
,172 mm2 bothway
Provide 8 mm O bar
spacing of bar = Area of one bar x 1000 / required area in m 2 / m
= 50.24 x 1000 /172
=
292
mm
Provide 8 mm O bar @ 290 mm C/C both ways, top and bottom.
Ast =
346
mm2
Designed section,Elevation etc. are shown in fig.
Top slab may be designed as two-way slab as usual for a live load of 1.5 KN / m 2
Top slab : consider 1 m wide strip. Assume 150 mm thick slab.
lx =
4 + 0.15 = 4.15 say 4.5 m
ly =

6 + 0.15 = 6.15 say 6.5 m


Dead Load : self 0.15 x 25 =
3.75
floor finish =
1.0
Live load =
1.5
6.25
For 1 m wide strip

KN / m2
KN / m2
KN / m2
KN / m2

PU =

1.5 x 6.25
= 9.38
KN / m

ly / l x =

6.5 / 4.5
=
1.4
AS Per IS-456-2000,Four Edges Discontinuous,positive moment at mid-span.
x =
Table 26
0.085
y =
Mx =

x x w x l

0.056
My =

2
x

= 0.085 x 9.38 x 4.52


= 16.15
KNm
From Table 6-3 ,Q = 2.76

M / Q x b
= 16.15 x 10

y x w x lx2
= 0.056 x 9.38 x 4.52
= 10.64
KNm

drequired =

76.50

/ 2.76 x 1000
mm

dshort = 150 - 15(cover) - 5


= 130 > 76.50
mm
(O.K.)
dlong = 130 - 10 =
120
mm
Larger depth is provided due to deflection check.
Mu / b x d2 (short) = 16.15 x 106 / 1000 x 130 x 130
=
0.96
Pt = 50 1-1-(4.6 / fck) x (Mu / b x d2)
fy / fck
=

50 1-1-(4.6 / 20) x (0.96)


415 / 20

= 50 [(1-0.88) x 20 / 415 ]
= 0.29%
Ast (short) = 0.29 x 1000 x 130 / 100
=
377
mm2
Provide 10 mm O bar
spacing of bar = Area of one bar x 1000 / required area in m 2 /m
= 78.50 x 1000 /377
=
208
mm
Provide 10 mm O bar @ 210 mm c/c
= 374
mm2 .
Mu / b x d2 (long) = 10.64 x 106 / 1000 x 120 x 120
=
0.74

Pt = 50 1-1-(4.6 / fck) x (Mu / b x d2)


fy / fck
=

50 1-1-(4.6 / 20) x (0.74)


415 / 20

= 50 [(1-0.91) x 20 / 415 ]
= 0.22%
Ast (long) = 0.22 x 1000 x 120 / 100
=
264
mm2
Provide 8 mm O bar
spacing of bar = Area of one bar x 1000 / required area in m 2 / m
= 50.24 x 1000 /264
=
190
mm
Provide 8 mm O bar @ 190 mm c/c
= 264
mm2 .

4m

TABLE 9-6
Balanced Design Factors for members in bending
For M20 Grade Concrete Mix
Mild steel
HYSD bars
d/D
Pt
Pt
Q = M / bD2
Q = M / bD2
0.75
0.3
0.4
0.295
0.289
0.8

0.305

0.37

0.299

0.272

0.85
0.9

0.31
0.314

0.355
0.335

0.302
0.306

0.258
0.246

TABLE 9-5

Members in bending ( Cracked condition )


Coefficients for balanced design
st
Grade of
Grade of cbc
2
concrete
steel
k
j
N / mm
N / mm2
For members less than 225mm thickness and tension on liquid face
M20
Fe250
7
115
0.445
0.851
Fe415
7
150
0.384
0.872

Q
1.33
1.17

For members more than 225mm thickness and tension away from liquid face
M20
Fe250
7
125
0.427
0.858
1.28
Fe415
7
190
0.329
0.89
1.03

s lies outside the section

D/2

e=M/T

TABLE 9-3
Minimum Reinforcement for Liquid Retaining Structures
Thickness, mm

% of reinforcement

100

Mild Steel
0.3

HYSD bars
0.24

150
200
250

0.286
0.271
0.257

0.229
0.217
0.206

300
350
400
450 or more

0.243
0.229
0.214
0.2

0.194
0.183
0.171
0.16

8 O @ 190 c/c

10 O @ 210 c/c
150

150 Free board

3500

10 O @ 200 c/c -

shape

10 O @ 200 c/c -

shape

1500

150
150

8 O @ 290 c/c both ways top and bottom


1500

1 : 4 : 8 P.C.C.
150 450

6000

450 150

Elevation
1500
450
1000

16 O @ 130 c/c (a)


16 O @ 200 c/c (b)
10 O @ 200 c/c both faces (c)
8 O @ 130 c/c (d)
12 O @ 130 c/c (e)

4000

v
1000 v

(a)

(b)

(d)

(c)
(d)

( a ) 1000

vv

450
1500

1500
6000

450

450

Section A-A

Table 6-3
Limiting Moment of resistance factor Q lim, N / mm2
fck N / mm2

fy, N / mm2

15
20
25

250
2.22
2.96
3.70

415
2.07
2.76
3.45

500
2.00
2.66
3.33

550
1.94
2.58
3.23

30

4.44

4.14

3.99

3.87

Pt,bal
1.36
0.98
1.2
0.61

Design of Rectangular water tank CASE-2 ( L / B 2 )


Size of tank : 3.6 m x 8.0 m x 3.0 m high
(given)
Material
M20
Grade Concrete
(given)
Fe 415
Grade HYSD reinforcement
(given)
Solution :Size of tank : 3.6 m x 8.0 m x 3.0 m high
Volume = 3.6 x 8 x 3.0 x 10 3 =
86400 Litres
8
/
3.6
=
2.22
>2.
L/B=
The long walls are designed as vertical cantilevers from the base.
The short walls are designed as supported on long walls.
If thickness of long walls is 400 mm, the span of the short wall = 3.6 + 0.4 = 4.0 m.
bottom 1 m or H / 4 whichever is more is designed as cantilever.
H/4=
3.0 / 4 =
0.75
m
bottom h = 1 m will be designed as cantilever.
Moments and tensions :
Maximum B.M. in long walls at the base
3
= (1 / 6 ) x Yw x H
= ( 1 / 6 ) x 9.8 x 33
= 44.1
KNm.
Maximum ( - ve ) B.M. in short walls at support
2
= Yw x ( H - h ) x B / 12
= 9.8 x ( 3 - 1 ) x 42 / 12
= 26.13
KNm.
Maximum ( + ve ) B.M. in short walls at centre
2
= Yw x ( H - h ) x B / 16
= 9.8 x ( 3 - 1 ) x 42 / 16
= 19.60
KNm.
For bottom portion
Yw x H x h2 / 6
M=
= 9.8 x 3.0 x 1 / 6
=
4.90
KNm
Direct tension in long wall =

OR

Yw x H / 6 , whichever is greater
= 9.8 x 3.0 / 6
=
4.90
KNm

Yw x ( H - h ) x B / 2
= 9.8 x ( 3 - 1 ) x 3.6 / 2
=
35.28
KN

Direct tension in short wall=


=
=

Yw ( H - h ) x 1
9.8 x ( 3 - 1 ) x 1
19.6
KN

It is assumed that end one metre width of long wall gives direct tension to short walls.
Design of long walls : M(-)=
44.1
KNm
T=
35.28
KN
From Table 9-6
Assume d / D = 0.9

M / Q x b
= 44.1 x 10

( water face )
( perpendicular to moment steel )
Q = 0.306

D=

=
Take D =

379.6

/ 0.306 x 1000

mm,

400 mm

d = 400 - 25 - 8
= 367 mm

Ast for Moment


From Table 9-5 ,
Ast = M / st x j x d
= 44.1 x 10 6 / 150 x 0.872 x 367
= 918.68

mm2

Provide 16 mm O bar
spacing of bar =

Area of one bar x 1000 / required area in m 2 / m


= 200.96 x 1000 / 918.68
= 218.749
mm
Provide 16 mm O bar @ 200 mm c/c = 1005
mm2 .

Note : The design is made at the base. The moment reduces from base to top.For economy, the
reinforcement can be curtailed or the thickness of wall can also be reduced as we have done for
cantilever retaining wall.
From Table 9-3
Distribution steel = 0.171 % for 400 mm depth
As = ( 0.171 / 100 ) x 1000 x 400
= 684
mm2 .
on each face = 342

mm2 .

( 1 )

Steel required for direct tension


= T / st
= 35.28 x 103 / 150
= 235
mm2 . ( 2 )
From ( 1 ) and ( 2 ) , minimum steel is sufficient for resisting direct tension.
Provide 8 mm O bar
spacing of bar =
Area of one bar x 1000 / required area in m 2 / m
= 50.24 x 1000 / 342
= 146.901
mm
Provide 8 mm O bar @ 140 mm c/c on each face = 357
mm2
on each face

Design of short walls :At support


From Table 9-5
Ast1 for moment =

M=
T=

26.13
19.6

KNm
KN

M / st x j x d
= 26.13 x 10 6 / 150 x 0.872 x 367
= 544
mm2

Ast2 for direct tension = T / st


=
19.6 x 10 3 / 150
=
131
mm2
Total Ast1 + Ast2 =
544 + 131
=

675

mm2

Provide 12 mm O bar
spacing of bar =

Area of one bar x 1000 / required area in m 2 / m


= 113.04 x 1000 / 675
= 167.467
mm
Provide 12 mm O bar@160 mm c/c = 706
mm2.
1000
203.56
400 367
163.44

checking :
modular ratio m =

x=

280 / 3 x cbc
= 13.33

b x D2 / 2 + Ast ( m - 1 ) x d

= 203.56

b x D + ( m - 1 ) x Ast
( 1000 x 4002 / 2 ) + ( 706 x ( 13.33 - 1 ) x 367 )
( 1000 x 400 ) + ( ( 13.33 - 1 ) x 706 )
mm

D-x=
196.44 mm
d-x=
163.44 mm
AT = b x D + ( m - 1 ) x Ast
= 1000 x 400 + (13.33 - 1 ) x 706

408705 mm2

Ixx = ( 1 / 3 ) x b x ( x3 + ( D - x )3 ) + ( m - 1 ) x Ast x ( d - x )2
= ( 1 / 3 ) x 1000 x ( 203.563 + 196.443 ) + ( 13.33 - 1 ) x 706 x 163.442
=
5.34E+09
+
2.33E+08
4
=
5.57E+09 mm
fct = T / AT
= 19.6 x 10 3 / 408705
=
0.048 N / mm2
fcbt = M x ( d - x ) / Ixx
= 26.13 x 106 x 163.44 / 5.57 x 10 9
= 0.767
N / mm2
check :
( fct / ct ) + ( fcbt / cbt )
1
( 0.048 / 1.2 ) + ( 0.767 / 1.7 ) 1
From Table 9-2
0.04 + 0.4512 1
0.4912 1
.. ( O. K. )
At centre :
M=
T=
From Table 9-5
Ast1 for moment =

19.6
19.6

KNm
KN

M / st x j x d
= 19.6 x 10 6 / 150 x 0.872 x 367
= 408
mm2

Ast2 for direct tension = T / st


=
19.6 x 10 3 / 150
=
131
mm2
Total Ast1 + Ast2 =
408 + 131
=

539

Provide 12 mm O bar
spacing of bar =

mm2

Area of one bar x 1000 / required area in m 2 / m


= 113.04 x 1000 / 539
= 209.722
mm
Provide 12 mm O bar @ 200 mm c/c = 565
mm2.

From Table 9-3


Distribution steel = 0.171 % for 400 mm depth

As = ( 0.171 / 100 ) x 1000 x 400


= 684
mm2 .
on each face = 342
mm2 .

( 1 )

Steel required for direct tension


= T / st
= 19.6 x 103 / 150
= 131
mm2 . ( 2 )
From ( 1 ) and ( 2 ) , minimum steel is sufficient for resisting direct tension.
Provide 8 mm O bar
spacing of bar =
Area of one bar x 1000 / required area in m 2 / m
= 50.24 x 1000 / 342
= 146.901
mm
Provide 8 mm O bar @ 140 mm c/c on each face = 357
mm2
on each face
Bottom cantilever
M = 4.9
KNm
From Table 9-5
Ast = M / st x j x d
= 4.9 x 10 6 / 150 x 0.872 x 367
= 102
mm2
Minimum steel = 342 mm2 on each face.
Provide 8 mm O bar @ 140 mm c/c on each faces = 357
mm2
on each face
Base slab :Base slab is resting on ground. For a water head 3 m, provide 150 mm thick slab.
From table 9-3
Minimum steel =
0.229%
= 0.229 / 100 x 1000 x 150
= 344
mm2
,172 mm2 bothway
Provide 8 mm O bar
spacing of bar =
Area of one bar x 1000 / required area in m 2 / m
= 50.24 x 1000 /172
= 292
mm
Provide 8 mm O bar @ 290 mm C/C both ways, top and bottom.
Ast =
346
mm2
Designed section,Elevation etc. are shown in fig.
Top slab may be designed as a one-way slab as usual for a live load of 1.5 KN / m 2
Top slab : consider 1 m wide strip. Assume 150 mm thick slab.
lx =
3.6 + 0.4 = 4 say 4 m

ly =

8 + 0.15 = 8.15 say 8.5 m


Dead Load : self 0.15 x 25 =
floor finish =
Live load =

3.75
1.0
1.5
6.25

KN / m2
KN / m2
KN / m2
KN / m2

For 1 m wide strip


PU =

1.5 x 6.25
=
9.38

KN / m

Maximum moment = 9.38 x 42 / 8


= 18.76
KNm
Maximum shear = 9.38 x 3.6 / 2
= 16.88
KN
From Table 6-3 ,Q = 2.76

M / Q x b
= 18.76 x 10

drequired =

82.44

/ 2.76 x 1000
mm

dprovided =

150 - 15(cover) - 6
= 129 > 82.44
(O.K.)
Larger depth is provided due to deflection check.
Design for flexure :
Mu / b x d2 = 18.76 x 106 / 1000 x 129 x 129
= 1.13
Pt = 50 1-1-(4.6 / fck) x (Mu / b x d2)
fy / fck
=

50 1-1-(4.6 / 20) x (1.13)


415 / 20

= 50 [(1-0.86) x 20 / 415 ]
= 0.34%
Ast = 0.34 x 1000 x 129 / 100
= 439
mm2
Provide 8 mm O bar
spacing of bar =
Area of one bar x 1000 / required area in m 2 /m
= 50.24 x 1000 /439
= 114
mm
Provide 8 mm O bar @ 110 mm c/c = 457
mm2 .

Minimum steel is 0.15 % for mild steel and 0.12 % for HYSD Fe415 reinforcement
Distribution steel = ( 0.12 / 100 ) x 1000 x 150
= 180
mm2
Provide 6 mm O bar
spacing of bar =

Area of one bar x 1000 / required area in m 2 / m


= 28.26 x 1000 /180
= 157
mm
Provide 6 mm O bar @ 150 mm c/c = 188
mm2 .

4.0 m.

hort walls.

op.For economy, the


as we have done for

42

Table 9-2
Permissible concrete stresses in calculations relating to
resistance to cracking
Grade of
concrete
M15
M20
M25
M30
M35
M40

Permissible stresses in N / mm2


Direct
Tension due to Shear stress
v = V / b j d
tension ct
bending cbt
1.1
1.5
1.5
1.2
1.7
1.7
1.3
1.5
1.6
1.7

1.8
2.0
2.2
2.4

1.9
2.2
2.5
2.7

6 O @ 150 c/c

8 O @ 110 c/c
150

12 O @ 200 c/c
8 O @ 140 c/c shape
8 O @ 140 c/c
8 O @ 290 c/c both ways top and bottom
2000

3000

150
150

2000

150

1 : 4 : 8 P.C.C.
150 400

8000

400

Section A-A

2000

8 O @ 140 mm c/c

400
900

16 O @ 200 c/c ( chipiya )

12 O @ 160 c/c(chipiya)
8 O @ 140 c/c (chipiya)

3600

8 O @ 140 c/c

900
400

12 O @ 200 c/c
900

v
v
2000

vv

2000

B
8000

400

Sectional plan

8 O @ 110 c/c
150

400

6 O @ 150 c/c

16 O @ 200 c/c ( chipiya )


8 O @ 140 c/c

3000

900

900

Base details not shown


for clarity

150
150
150 400

3600
Section B- B

400 150

Design of simply supported one way slab


effective span = 4 m supported on masonry wall of 230 mm thickness
Live load =
2.5 KN / m2 ( given )
Floor finish =
1 KN / m2 ( given )
material M15 grade concrete
( given )
HYSD reinforcement grade Fe415
( given )
solution : Assume 0.4 % steel , a trial depth by deflection criteria
IS 456-2000 clause 23.2.1 fig-4 ,for tension reinforcement
modification factor = 1.26
for simply supported, basic span / effective depth ratio = 20
( span / d ) ratio permissible = 1.26 x 20
= 25.2
drequired = 4000 / 25.2
= 158.7
mm
D = 158.7 + 15 ( cover ) + 5 ( assume 10 O bar )
= 178.7
mm
Assume an overall depth = 180
mm
2
DL = 0.18 x 25 = 4.5
KN / m
Floor finish = 1.0
KN / m2
Live load = 2.5
KN / m2
Total 8.0
KN / m2
Factored load = 1.5 x 8 = 12
KN / m
Consider 1 m length of slab
Maximum moment = w x l2 / 8
= 12 x 42 / 8
= 24
KNm
Maximum shear = w x l / 2
= 12 x 4 / 2
= 24
KN
Design for flexure : d = 180 - 15 - 5
= 160
mm
Mu / b x d2 = 24 x 10 6 / 1000 x (160)2
= 0.94
Pt = 50 1-1-(4.6 / fck) x (Mu / b x d2)
fy / fck
= 50 1-1-(4.6 / 15) x (0.94)
415 / 15
= 50 [(1-0.84) x 15 / 415 ]

( given )

= 0.289%
Ast = 0.289 x 1000 x 160 / 100
= 462
mm2
Provide 10 mm O bar
spacing of bar = Area of one bar x 1000 / required area in m 2 / m
= 78.50 x 1000 /462
=
170
mm
Provide 10 mm O bar @ 170 mm c/c = 462
mm2 .
Half the bars are bent at 0.1 l = 400 mm , and remaining
bars provide 231 mm2 area
100 x As / ( b x D ) = 100 x 231 / ( 1000 x 180 )
= 0.128
> 0.12 % ( minimum steel for Fe415)
i.e. remaining bars provide minimum steel. Thus, half the bars may be bent up.
Distribution steel = ( 0.12 /100 ) x 1000 x 180
= 216
mm2
Maximum spacing = 5 x 160 = 800 or 450 mm
i.e. 450 mm
Provide 8 mm O bar
spacing of bar = Area of one bar x 1000 / required area in m 2 /m
= 50.24 x 1000 /216
= 233
mm
Provide 8 mm O bar @ 230 mm c/c = 218
mm2 .
Check for shear : Vu = 24
KN
Actual Shear stress =

Vu / b x d

= 24 x 103 / 1000 x 160


= 0.150

N / mm2

< (

) N / mm2 ( too small )

For bars at support


d = 160
mm
As = 231
mm2 .
100 x As / b x d = 100 x 231 / 1000 x 160
= 0.144
Design shear strength c = 0.85 0.8 x fck ( 1 + 5 x - 1 )
6x
= 0.8 x fck / 6.89 Pt , but not less than 1.0
= 0.8 x 15 / 6.89 x 0.144
= 12.1
Design shear strength c = 0.85 0.8 x 15 ( 1 + 5 x 12.1 - 1 )
6 x 12.1
= 0.277
IS 456-2000 Table 19 from table 7-1

for Pt = 0.144
c = 0.28 N / mm2
IS 456-2000 clause 40.2.1.1
25 difference -0.05
k = 1.24
for 180 mm slab depth
20 difference
?
Design shear strength = 1.24 x 0.28
= 0.347
N / mm2 .( O.K.)
Check for development length : Assuming L0 = 8 O
(HYSD Fe415 steel ) For continuing bars
Mu1 =
=
=
=

As = 231
mm2
0.87 x fy x Ast x d { 1 - ( fy x Ast / fck x b x d ) }
0.87 x 415 x 231 x 160 { 1 - (415 x 231 / 15 x 1000 x 160 ) }
13.34 { 1- 0.0399 }
12.812 KNm

-0.04

OR

Vu = 24

KN
Development length of bars Ld = O s / 4 x bd

= 56 O ( from Table 7-6 )

1.3 x ( Mu1 / Vu ) + L0 Ld

1.3 x ( 12.81 x 106 / 24 x 103 ) + 8 O 56 O


693.875 + 8 O
56 O
48 O 693.88
which gives
O 14.46
mm
.( O.K.)
Check for deflection : Basic ( span / d ) ratio = 20
Pt = 100 x Ast / b x d = 100 x 462 / 1000 x 160
= 0.289
IS 456-2000 clause 23.2.1 fig-4 ,for tension reinforcement
modification factor = 1.42
( span / d ) ratio permissible = 1.42 x 20
= 28.4
Actual (span / d ) ratio = 4000 / 160
= 25.00
<
28.4
.( O.K.)
The depth could be reduced
Check for cracking : IS 456-2000 , clause 26.3.3
Main bars : maximum spacing permitted = 3 x effective depth of slab or 300 mm whichever is sm
= 3 x 160 = 480
mm
or 300 mm i.e. 300 mm
spacing provided = 170
mm
< 300 mm
.( O.K.)
Distribution bars : maximum spacing permitted = 5 x effective depth of slab or 450 mm whiche
= 5 x 160 = 800 mm
spacing provided = 230
mm
< 450 mm
.( O.K.)

For tying the bent bars at top , provide 8 mm O @ 230 mm c/c

NOTE : If clear span = 3.77 m , effective span = 3.77 + 0.23 = 4 m


effective span = 3.77 + 0.16 ( effective depth ) = 3.93 m

OR

0.6 % for mild steel reinforcement and 0.3 to 0.4 % for HYSD Fe415 grade
reinforcement
NOTE : -

For mild steel minimum reinforcement 0.15 %

For checking development length , l0


may be assumed as 8 O for HYSD
bars ( usually end anchorage is not
provided ) and 12 O for mild steel ( U
hook is provided usually whose
anchorage length is 16 O.

Pt = 100 x As / b x d
= 100 x 231 / 1000 x 160
= 0.14
From equation
Pt = 50 1-1-(4.6 / fck) x (Mu / b x d2)
fy / fck
Mu1 / b x d2 = 0.49
Mu1 = 0.49x 1000 x 1602 x 10-6

we get ,

= 12.54

KNm

160
180
400

r 300 mm whichever is small


or 300 mm i.e. 300 mm
.( O.K.)
h of slab or 450 mm whichever is small
or 450 mm i.e. 450 mm
.( O.K.)

v
v

v
8 O @ 230 c/c
v
10 O @ 170 c/c
( alternate bent )
4000

v
v
400

v
v

If ly / lx 2 ,called one way slab provided that it


is supported on all four edges . Note that , if all
four edge is not supported and ly / lx < 2 , then
also it is one-way slab,If ly / lx < 2 , called twoway slab.provided that it is supported on all
four edges.
IS 456-2000 clause 22.2
Effective Span
( a ) Simply Supported Beam or Slab The effective span of a member that is not built integrally with its
supports shall be taken as clear span plus the effective depth of
slab or beam or centre to centre of supports , whichever is less.
( b )Continuous Beam or Slab - In the case of
continuous beam or slab, if the width of the
support is less than l/12 of the clear span, the
effective span shall be as in (a). If the
supports are wider than I/12 of the clear span
or 600 mm whichever is less, the effective span
shall be taken as under:
1) For end span with one end fixed and the
other continuous or for intermediate spans,
the effective span shall be the clear span
between supports;
2) For end span with one end free and the other
continuous, the effective span shall be equal
to the clear span plus half the effective depth
of the beam or slab or the clear span plus
half the width of the discontinuous support,
whichever is less;
3) In the case of spans with roller or rocket
bearings, the effective span shall always be
the distance between the centres of bearings.
( c )Cantilever-The effective length of a cantilever
shall be taken as its length to the face of the
support plus half the effective depth except
where it forms the end of a continuous beam
where the length to the centre of support shall
be taken.
( d )Frames-In the analysis of a continuous frame,
centre to centre distance shall be used.

elopment length , l0
as 8 O for HYSD
anchorage is not
O for mild steel ( U
usually whose
is 16 O.

Design of Cantilever one way slab


used for residential purpose
at the free end of slab S1 ,concrete parapet of 75 mm thick and 1 m high.
material M15 grade concrete
mild steel grade Fe250
Live Load As per IS 875
Solution : For slab S2 live load = 2 KN /m2

( given )
( given )

For slab S1 live load = 3 KN /m2

( Balcony slab )
DL
LL

Assume 120 mm thick slab


For S2
self load = 0.12 x 25 = 3
floor finish = 1
live load = 0
Total
4
Pu = 1.5 ( 4 + 2 ) = ( 6 + 3 )
DL
For S1
self load = 0.12 x 25 = 3
floor finish = 1
live load = 0
Total
4

0
0
2
2

ly = 6m

KN / m2
KN / m2
KN / m2
KN / m2

KN / m2
LL
0
0
3
3

KN / m2
KN / m2
KN / m2
KN / m2

Pu = 1.5 ( 4 + 3 ) = (6 + 4.5) KN / m2
Weight of parapet 0.075 x 25 x 1 = 1.875
KN / m
Pu = 1.5 x 1.875 = 2.8
KN / m
Consider 1 m long strip
(1) To get maximum positive moment in slab S2 only dead load on slab
S1 and total load on slab S2 shall be considered
9 KN/m

6 KN/m

SS22
A

3m

S1
B

1.2m

(a) Loads for maximum positive moment


9 KN/m

10.5 KN/m

S2
A

3m

2.8 KN

S1
B

1.2m

(b) Loads for maximum negative


moment,maximum shear for cantilever span
and maximum reaction at support B
considering fig (a)
cantilever moment = wx l2 / 2

= 6 x 1.22 / 2
= 4.32
KNm
shear =
Reaction at A = w x l / 2 - Moment @ B at distance 3 m
= 9 x 3 / 2 - 4.32 / 3
= 12.06
KN
Point of zero shear from A = 12.06 ( KN ) / 9 ( KN / m )
= 1.34
m
Maximum positive moment = 12.06 x 1.34 - W x l2 / 2
= 12.06 x 1.34 - 9 x 1.342 / 2
= 8.08
KNm
(2) To get maximum negative moment and maximum shear at B,
the slab is loaded with full loads as shown in fig (b)
Maximum negative moment = w x l + w x l2 / 2
= 2.8 x 1.2 + 10.5 x 1.22 / 2
= 10.92
KNm
Maximum shear at B
Vu,BA = w x l / 2 + Moment @ B at distance 3 m
= 9 x 3 / 2 + 10.92 / 3
= 17.14
KN
Vu,BC = w x l + 2.8
= 10.5 x 1.2 + 2.8
= 15.4
KN
Moment steel :
Maximum moment = 10.92
KNm
From Table 6-3
Q = 2.22

M / Q x b
= 10.92 x 10

drequired =

=
dprovided =
=
Mu / b x d2 ( + ) =
=

70.14

/ 2.22 x 1000

mm,

120 - 15 - 6 ( assume 12 O bar )


99
mm,
.( O.K.)
8.08 x 10 6 / 1000 x (99)2
0.82

Pt = 50 1-1-(4.6 / fck) x (Mu / b x d2)


fy / fck
= 50 1-1-(4.6 / 15) x (0.82)
250 / 15
= 50 [(1-0.865) x 15 / 250 ]
= 0.405%

Ast ( + ) = 0.405 x 1000 x 99 / 100


= 401
mm2
Provide 10 mm O bar
spacing of bar = Area of one bar x 1000 / required area in m 2 / m
= 78.5 x 1000 / 401
= 195.761
mm
Provide 10 mm O bar@170 mm c/c = 462
mm2. ( alternate bent up )
Mu / b x d2 ( - ) = 10.92 x 10 6 / 1000 x (99)2
= 1.11
Pt = 50 1-1-(4.6 / fck) x (Mu / b x d2)
fy / fck
= 50 1-1-(4.6 / 15) x (1.11)
250 / 15
= 50 [(1-0.812) x 15 / 250 ]
= 0.564%
Ast ( - ) = 0.564 x 1000 x 99 / 100
= 558
mm2
Provide 10 mm O bar
Provide 10 mm O bar@340 mm c/c = 231
mm2. ( bent bars extended )
Area provided = Area of one bar x 1000 / spacing of bar in m
= 78.5 x 1000 / 340
= 231 mm2
remaining area = 558 - 231
= 327
mm2
Provide 12 mm O bar
spacing of bar = Area of one bar x 1000 / required area in m 2 / m
= 113.04 x 1000 / 327
= 346
mm
Provide 12 mm O bar@340 mm c/c = 332
mm2. ( Extra )
Total 231 + 332 mm2 = 563 mm2 steel provided
Note that at simple support , the bars are bent at 0.1 l whereas at continuity of slab it
is bent at 0.2 l
For mild steel minimum reinforcement 0.15 %
Distribution steel = ( 0.15 /100 ) x 1000 x 120
= 180
mm2.
Provide 6 mm O bar
spacing of bar = Area of one bar x 1000 / required area in m 2 / m
= 28.26 x 1000 / 180
= 157
mm

Provide 6 mm O bar@150 mm c/c = 188


mm2.
For negative moment reinforcement
Development length of bars Ld = O s / 4 x bd
( from Table 7-6 )
= O x 0.87 x 250 / 4 x 1
= 54.4
O
Ld = 54.4 x ( 10 + 12 ) / 2
= 598
mm.
say 600 mm
As a thumb rule, a bar shall be given an anchorage equal to the length of
the cantilever.
Check for development length : Assuming L0 = 12 O
(mild steel )
Pt = 100 x As / b x d
= 100 x 231 / 1000 x 99
= 0.233
From equation
At A ,

Half bars bent = 462 / 2 = 231 mm2 )

OR

Pt = 50 1-1-(4.6 / fck) x (Mu / b x d2)

Mu1 = 0.87 x fy x Ast x d ( 1 - fy x

fy / fck
we get ,

= 0.87 x 250 x 231 x 99 ( 1

Mu1 / b x d2 = 0.487
Mu1 = 0.487 x 1000 x 992 x 10-6
= 4.77
KNm
Vu = 12.06
KN

Development length of bars Ld =

O s / 4 x bd

= 4.97401
= 4.78

= O x 0.87 x 250 / 4 x 1

=54.4 O

1.3 x ( Mu1 / Vu ) + L0 Ld
1.3 x ( 4.77 x 106 / 12.06 x 103 ) + 12 O 54.4 O
514.179 + 12 O 54.4 O
42.4 O 514.179
which gives
O 12.13
mm
.( O.K.)
Pt = 100 x As / b x d
At B ,
Half bars bent = 462 / 2 = 231 mm2 )
= 100 x 231 / 1000 x 99
= 0.233
From equation
OR
Pt = 50 1-1-(4.6 / fck) x (Mu / b x d2)

Mu1 = 0.87 x fy x Ast x d ( 1 - fy x

fy / fck

= 0.87 x 250 x 231 x 99 ( 1

Mu1 / b x d2 = 0.487
Mu1 = 0.487 x 1000 x 992 x 10-6
= 4.77
KNm
Near point of contraflexure i.e. 0.15 x l from B
Vu = 17.14 - ( 0.15 x 3 ) x 9 =
we get ,

Development length of bars Ld =

O s / 4 x bd

4.974

= 4.78

13.09

KN

= O x 0.87 x 250 / 4 x 1

=54.4 O

1.3 x ( Mu1 / Vu ) + L0 Ld
1.3 x ( 4.77 x 106 / 13.09 x 103 ) + 12 O 54.4 O
473.72 + 12 O 54.4 O
42.4 O 473.72
which gives
O 11.2
mm
Check for shear : Span AB :
At A , Vu,AB = 12.06 KN ( for maximum loading )
At B , shear at point of contraflexure = 13.09
Use Vu = 13.09 KN

.( O.K.)

KN

Shear stress v = Vu / b x d
= 13.09 x 10 3 / 1000 x 99
= 0.132
N / mm2 < c

.( O.K.)

100 x As / b x d = 100 x 231 / 1000 x 99


= 0.233
Design shear strength c = 0.85 0.8 x fck ( 1 + 5 x - 1 )
6x
= 0.8 x fck / 6.89 Pt , but not less than 1.0
= 0.8 x 15 / 6.89 x 0.233
= 7.47
Design shear strength c = 0.85 0.8 x 15 ( 1 + 5 x 7.47 - 1 )
6 x 7.47
= 0.34
IS 456-2000 Table 19 from table 7-1
for Pt = 0.233
c = 0.34 N / mm2
IS 456-2000 clause 40.2.1.1
0.1 difference 0.07
k = 1.3
for 120 mm slab depth
0.02 difference
?
0.014
Design shear strength = 1.3 x 0.34
= 0.442
N / mm2 > v
.( O.K.)
Span BC :
Vu = 17.14 KN
Pt =

Shear stress v = Vu / b x d
= 17.14 x 10 3 / 1000 x 99
= 0.173
N / mm2 < c

.( O.K.)

100 x As / b x d = 100 x 563 / 1000 x 99


= 0.569
Design shear strength c = 0.85 0.8 x fck ( 1 + 5 x - 1 )
6x
= 0.8 x fck / 6.89 Pt , but not less than 1.0
= 0.8 x 15 / 6.89 x 0.569
Pt =

= 3.06
Design shear strength c = 0.85 0.8 x 15 ( 1 + 5 x 3.06 - 1 )
6 x 3.06
= 0.487
IS 456-2000 Table 19 from table 7-1
for Pt = 0.569
c = 0.48 N / mm2
IS 456-2000 clause 40.2.1.1
0.25 difference 0.08
k = 1.3
for 120 mm slab depth
0.181 difference
?
0.05792
Design shear strength = 1.3 x 0.48
= 0.624
N / mm2 > v
.( O.K.)
Check for deflection : For span AB :
Basic ( span / d ) ratio = 20
Pt = 100 x Ast / b x d = 100 x 462 / 1000 x 99
= 0.467
IS 456-2000 clause 23.2.1 fig-4 ,for tension reinforcement
modification factor = 2
( span / d ) ratio permissible = 2 x 20
= 40
Actual (span / d ) ratio = 3000 / 99
= 30.30
<
40
.( O.K.)
For span BC :
Basic ( span / d ) ratio = 7
Pt = 100 x Ast / b x d = 100 x 563 / 1000 x 99
= 0.569
IS 456-2000 clause 23.2.1 fig-4 ,for tension reinforcement
modification factor = 1.8
( span / d ) ratio permissible = 1.8 x 7
= 12.6
Actual (span / d ) ratio = 1200 / 99
= 12.12
<
12.6
.( O.K.)
Check for cracking : IS 456-2000 , clause 26.3.3
(1) Main bars : maximum spacing permitted = 3 x effective depth of slab or 300 mm whichever is sma
= 3 x 99
= 297
mm
or 300 mm i.e. 297 mm
spacing provided = 170
mm
< 297 mm
.( O.K.)
(2 )Distribution bars : maximum spacing permitted = 5 x effective depth of slab or 450 mm whiche
= 5 x 99
= 495 mm
spacing provided = 150
mm
< 450 mm
.( O.K.)

S3

1.2m
0.15m

ly = 6m

0.15m

Column 300 x 300

B3

B1

B2
S2

S1

B4

0.15m

0.15m
lx = 3m

1.2m

1m high parapet

0.87 x fy x Ast x d ( 1 - fy x Ast / b x d x fck )


0.87 x 250 x 231 x 99 ( 1 - 250 x 231 / 1000 x 99 x 15 ) x 10 -6
( 1 - 0.0389 )
KNm

0.87 x fy x Ast x d ( 1 - fy x Ast / b x d x fck )


0.87 x 250 x 231 x 99 ( 1 - 250 x 231 / 1000 x 99 x 15 ) x 10 -6
( 1 - 0.0389 )
KNm

10 O @ 340 c/c (bent)


+ 12 O @ 340 c/c (extra)
1200

1200

120

125
10 O @ 170 c/c

6 O @ 150 c/c

6 O @ 150 c/c
300
150

r 300 mm whichever is small


or 300 mm i.e. 297 mm
.( O.K.)
of slab or 450 mm whichever is small
or 450 mm i.e. 450 mm
.( O.K.)

600
3000

150

1200

Design of Continuous Two-way slab


Banking hall
slab is restrained with edge beams
Material M15 grade concrete
HYSD reinforcement of grade Fe415
Solution : Assume 150 mm thick slab
Self load 0.15 x 25 = 3.75
KN / m2
floor finish = 1.00
KN / m2
Live Load = 3.00
KN / m2
Total
7.75
KN / m2
Pu = 1.5 x 7.75

given

= 11.625 KN / m2
ly / lx = 5 / 5
=1
< 2
i.e. two-way slab.
Middle strip :
IS 456-2000 Table -26
Two adjacent edges Discontinuous
Mu1 , Mu3 ( - ) = x x w x lx2 =
0.047 x 11.625 x 52 =
Mu2 ( + ) = y x w x l =
0.035 x 11.625 x 52 =
From Table 6-3
For M15 mix and Fe415 steel, Q = 2.07
2
x

M / Q x b
= 13.66 x 10

13.66

KNm

10.17

KNm

drequired =

/ 2.07 x 1000

= 81.23
mm,
dprovided for positive moment reinforcement
= 150 - 15 - 10 - 5 ( assume 10 O bar )
= 120
mm,
> 81.23 mm
.( O.K.)
dprovided for negative moment reinforcement
= 150 - 15 - 5
= 130
mm,
> 81.23 mm
.( O.K.)
Mu / b x d2 ( + ) = 10.17 x 10 6 / 1000 x (120)2
= 0.706
Pt = 50 1-1-(4.6 / fck) x (Mu / b x d2)
fy / fck
= 50 1-1-(4.6 / 15) x (0.706)
415 / 15
= 50 [(1-0.885) x 15 / 415 ]
= 0.208%

Ast = 0.208 x 1000 x 120 / 100


= 250
mm2
Provide 8 mm O bar
spacing of bar = Area of one bar x 1000 / required area in m 2 / m
= 50.24 x 1000 / 250
= 200.96
mm
Provide 8 mm O bar@ 200 mm c/c = 251
mm2.
Mu / b x d2 ( - ) = 13.66 x 10 6 / 1000 x (130)2
= 0.81
Pt = 50 1-1-(4.6 / fck) x (Mu / b x d2)
fy / fck
= 50 1-1-(4.6 / 15) x (0.81)
415 / 15
= 50 [(1-0.867) x 15 / 415 ]
= 0.240%
Ast = 0.24 x 1000 x 130 / 100
= 312
mm2
Provide 8 mm O bar
spacing of bar = Area of one bar x 1000 / required area in m 2 / m
= 50.24 x 1000 / 312
= 161
mm
Provide 8 mm O bar@ 150 mm c/c = 335
mm2.
For HYSD Fe415
Minimum steel = ( 0.12 / 100 ) x 1000 x 150
= 180
mm2
At discontinuous edges 4 and 5 , 50 % of the positive steel is required at top
= ( 1 / 2 ) x 251
= 126
mm2
This is less than minimum , therefore , use minimum steel at location 4 and 5 .
Provide 8 mm O bar
spacing of bar = Area of one bar x 1000 / required area in m 2 / m
= 50.24 x 1000 / 180
= 279.111
mm
Provide 8 mm O bar@ 260 mm c/c = 193
mm2.
More steel is provided to match with the torsion reinforcement.
In edge strip , minimum reinforcement is provided equal to 8 mm O @ 260 mm c/c.
Torsion steel :At corner A ,
steel required = ( 3/4 ) x 250

= 188

mm2.

Provide 8 mm O bar
spacing of bar = Area of one bar x 1000 / required area in m 2 / m
= 50.24 x 1000 / 188
= 267.234
mm
Provide 8 mm O bar@ 260 mm c/c = 193
mm2.
This will be provided by minimum steel of edge strip,
At corner B ,
steel required = ( 1/2 ) x 188
= 94
mm2.
Provide 8 mm O bar
spacing of bar = Area of one bar x 1000 / required area in m 2 / m
= 50.24 x 1000 / 180
= 279.111
mm
Provide 8 mm O bar@ 260 mm c/c = 193
mm2.
This will be provided by minimum steel .
Note that positive reinforcements are not curtailed because if they are curtailed , the
remaining bars do not provide minimum steel.
Check for shear : At point 1 or 3
S.F. = w x l / 2 + Moment @ point 1 or 3 in that span
= 11.625 x 5 / 2 + 13.66 / 5
= 31.795 KN
100 x As / b x d = 100 x 335 / ( 1000 x 130 )
= 0.258
from table 7-1
for Pt = 0.258 , c = 0.354 N / mm2
0.11
0.25 differen
IS 456-2000 clause 40.2.1.1
?
-0.1065
0.242 differe
k = 1.3
for 150 mm slab depth
Design shear strength = 1.3 x 0.354
= 0.460
N / mm2 .( O.K.)
OR
Design shear strength c = 0.85 0.8 x fck ( 1 + 5 x - 1 )
6x
= 0.8 x fck / 6.89 Pt , but not less than 1.0
= 6.75
Design shear strength c = 0.85 0.8 x 15 ( 1 + 5 x 6.75 - 1 )
6 x 6.75
=
0.356
IS 456-2000 clause 40.2.1.1
k = 1.3
for 150 mm slab depth
Design shear strength = 1.3 x 0.356
= 0.463
N / mm2 .( O.K.)
Actual shear stress = Vu / b x d

= 31.795 x 103 / ( 1000 x 130 )


= 0.245
N / mm2 < c
.( O.K.)
At point 4 or 5
S.F. = w x l / 2
= 11.625 x 5 / 2
= 29.06
KN
100 x As / b x d = 100 x 251 / ( 1000 x 120 )
= 0.209
from table 7-1
for Pt = 0.209 , c = 0.321 N / mm2
0.1 differenc
IS 456-2000 clause 40.2.1.1
0.041 differe
k = 1.3
for 150 mm slab depth
Design shear strength = 1.3 x 0.321
= 0.417
N / mm2
OR
Design shear strength c = 0.85 0.8 x fck ( 1 + 5 x - 1 )
6x
= 0.8 x fck / 6.89 Pt , but not less than 1.0
= 8.33
Design shear strength c = 0.85 0.8 x 15 ( 1 + 5 x 8.33 - 1 )
6 x 8.33
= 0.326
IS 456-2000 clause 40.2.1.1
k = 1.3
for 150 mm slab depth
Design shear strength = 1.3 x 0.326
= 0.424
N / mm2
Actual shear stress = Vu / b x d

0.07
?

-0.0287

= 29.06 x 103 / ( 1000 x 120 )


= 0.242
N / mm2 < c
.( O.K.)
Check for development length : This is critical at point 4 or 5.
Vu = 29.06
At point 4 or 5 ,
KN
No bar is curtailed or bent up.
Assuming L0 = 8 O
(HYSD Fe415 steel )
Pt = 100 x As / b x d
= 100 x 251 / 1000 x 120
= 0.209
From equation
Pt = 50 1-1-(4.6 / fck) x (Mu / b x d2)
fy / fck
we get ,

Mu1 / b x d2 = 0.711

OR
Mu1 = 0.87 x fy x Ast x d ( 1 - fy
= 0.87 x 415 x 251 x 120
= 10.8748

Mu1 = 0.711 x 1000 x 1202 x 10-6


= 10.246
= 10.24
KNm
Development length of bars Ld = O s / 4 x bd
=54.3 O (From Table 7-6 )
1.3 x ( Mu1 / Vu ) + L0 Ld
1.3 x ( 10.246 x 106 / 29.06 x 103 ) + 8 O 54.3 O
458.355 + 8 O
54.3 O
46.3 O 458.355
which gives
O 9.90
mm
.( O.K.)
Vu = 11.25 x ( 4.5 / 2 ) = 25.31 KN
Short span
Assuming L0 = 8 O
(HYSD Fe415 steel )
Pt = 100 x As / b x d
= 100 x 462 / 1000 x 160
= 0.289
From equation

OR

Pt = 50 1-1-(4.6 / fck) x (Mu / b x d )


2

fy / fck

Mu1 = 0.87 x fy x Ast x d ( 1 - fy


= 0.87 x 415 x 462 x 160

Mu1 / b x d = 0.9595
= 26.6888
Mu1 = 0.9595 x 1000 x 1602 x 10-6
= 24.5539
= 24.56
KNm
Development length of bars Ld = O s / 4 x bd
=56 O (From Table 7-6 )
1.3 x ( Mu1 / Vu ) + L0 Ld
we get ,

1.3 x ( 24.56 x 106 / 25.31 x 103 ) + 8 O 56 O


1261.48 + 8 O
56 O
48 O 1261.48
which gives
O 26.28
mm
.( O.K.)
Note that the bond is usually critical along long direction.
Check for deflection : Basic ( span / d ) ratio = 26
positive moment steel = 251
mm2.
actual d = 150 - 15 -8 - 4
= 123
mm.
Pt = 100 x Ast / b x d = 100 x 251 / 1000 x 123
240.7
= 0.204
IS 456-2000 clause 23.2.1 fig-4 ,for tension reinforcement
modification factor = 1.65
( span / d ) ratio permissible = 1.65 x 26
= 42.9
Actual (span / d ) ratio = 5000 / 123
= 40.65
<
42.9
.( O.K.)
Check for cracking : IS 456-2000 , clause 26.3.3

(1) Main bars : maximum spacing permitted = 3 x effective depth of slab or 300 mm whiche
= 3 x 130
= 390
mm
spacing provided = 200
mm
< 300 mm
.( O.K.)
(2) Distribu. bars : maximum spacing permitted = 5 x effective depth of slab or 450 mm whiche
= 5 x 120
= 600
mm
spacing provided = 260
mm
< 450 mm
.( O.K.)
Note that the bottom reinforcements are both ways and therefore there is no necessity
of secondary reinforcements.However , top reinforcement in edge strip requires the
secondary steel for tying the bars.

B
625

3750

625

625
A
3750

S1

625

Edge strip
8 O @ 260 c/c

Middle Strip
8 O @ 200 c/c

Edge strip
8 O @ 260 c/c

B
500

8 O @ 200 c/c

8 O @ 150 c/c
8 O @ 260 c/c

150

8 O @ 260 c/c
1500

1500
625

3750

Section A-A

625

8 O @ 200 c/c

Edge strip Middle Strip Edge strip


8 O @ 260 c/c
8 O @ 260 c/c

This is minimum (0.12 / 100) x 150 x 1000 = 180


mm2
Provide 8 mm O bar
spacing of bar = Area of one bar x 1000 / required area in m 2 / m
= 50.24 x 1000 / 180
= 279.111
mm
Provide 8 mm O bar@ 260 mm c/c = 193
mm2. for uniformity in spacing.
For clarity , top and bottom reinforcements are shown separately.

5m

5m

-0.047

S1
1

S1
Middle Strip

3750

625

ly/8

(3/4)ly

ly/8

A
625

-0.047

0.035

Edge strip

5m

Edge strip

0.035

4
5m

0.87 x fy x Ast x d ( 1 - fy x Ast / b x d x fck )


0.87 x 415 x 251 x 120 ( 1 - 415 x 251 / 1000 x 120 x 15 ) x 10
( 1 - 0.0579 )

-6

KNm

0.87 x fy x Ast x d ( 1 - fy x Ast / b x d x fck )


0.87 x 415 x 462 x 160 ( 1 - 415 x 462 / 1000 x 160 x 15 ) x 10
( 1 - 0.0799 )
KNm

-6

1000
625

625

625

1500

1500

500
3750

1500

S1

625

1500

1000

3750

Edge strip Middle Strip Edge strip


8 O @ 240 c/c
8 O @ 240 c/c
8 O @ 140 c/c

of slab or 300 mm whichever is small


or 300 mm i.e. 300 mm
.( O.K.)
of slab or 450 mm whichever is small
or 450 mm i.e. 450 mm
.( O.K.)

Edge strip
8 O @ 240 c/c

Middle Strip
8 O @ 140 c/c

8 O @ 240 c/c

Edge strip
8 O @ 240 c/c

8 O @ 140 c/c
500

150
8 O @ 180 c/c
1500

8 O @ 240 c/c
1500
625

3750

625

8 O @ 240 c/c

-0.047
3

Design of Continuous One-way slab


A five span continuous one-way slab used as an office floor.
The centre-to-centre distance of supporting beams is 3 m
Live load 3 KN / m2 and floor finish 1 KN / m2
Material M15 grade concrete
HYSD reinforcement of grade Fe415
Solution : Try 120 mm thick slab
DL
LL
Dead load 0.12 x 25 = 3
0
KN / m2
floor finish = 1
0
KN / m2
live load = 0
3
KN / m2
Total
4
3
KN / m2
factored load = 1.5 ( 4 + 3 )
= ( 6 + 4.5 ) KN / m2
Consider 1 m wide strip of the slab.

given

( 6 + 4.5 ) KN / m

1
A

3m

2
B

3m C

3m

2
D

3m

1
E

3m

The factored moments at different points using the coefficients are as follows :
Mu1 ( + ) = ( 1 / 12 ) x w ( DL ) x l2 + ( 1 / 10 ) x w ( LL ) x l2
=
=
=
Mu2 ( + ) =

( 1 / 12 ) x 6 x 32 + ( 1 / 10 ) x 4.5 x 32
4.5
+
4.05
8.55
KNm

=
=
=
Mu4 ( - ) =

( 1 / 10 ) x 6 x 32 + ( 1 / 9 ) x 4.5 x 32
5.4
+
4.5
9.9
KNm

( 1 / 16 ) x w ( DL ) x l2 + ( 1 / 12 ) x w ( LL ) x l2
= ( 1 / 16 ) x 6 x 32 + ( 1 / 12 ) x 4.5 x 32
= 3.38
+
3.38
= 6.75
KNm
Mu3 ( - ) = ( 1 / 10 ) x w ( DL ) x l2 + ( 1 / 9 ) x w ( LL ) x l2

( 1 / 12 ) x w ( DL ) x l2 + ( 1 / 9 ) x w ( LL ) x l2
= ( 1 / 12 ) x 6 x 32 + ( 1 / 9 ) x 4.5 x 32
= 4.50
+
4.50
=9
KNm
Maximum shear is Vu(BA) = 0.6 x w x l + 0.6 x w x l
= 0.6 x 6 x 3 + 0.6 x 4.5 x 3
= 18.9
KN
Maximum moment is Mu3 ( - ) = 9.9 KNm

From Table 6-3

Q = 2.07

M / Q x b
= 9.9 x 10

drequired =

/ 2.07 x 1000
mm,
6

= 69.2
Try 110 mm overall depth
dprovided = 110 - 15 - 5 ( assume 10 O bar )
= 90
mm,
.( O.K.)
Pt = 50 1-1-(4.6 / fck) x (Mu / b x d2)
fy / fck
Ast = Pt x b x d / 100
spacing of bar = Area of one bar x 1000 / required area in m 2 / m
Table
Factored
Ast
Pt
moment Mu/(b x d )
point
Steel Provided
KNm
mm2
2

1(+)

8.55

1.06

0.322

290

2(+)

6.75

0.83

0.248

223

3(-)

9.9

1.22

0.38

342

4(-)

9.0

1.11

0.34

306

10 mm O @ 440 c/c + 8 mm O @ 440 c/c


= 178 +114 = 292 mm2 (Half 10 O+half 8 O)

8 mm O @ 220 c/c
= 228 mm2
10 mm O @ 220 c/c
= 357 mm2
10 mm O @ 220 c/c
= 357 mm2

For Main steel ,HYSD Fe415 reinforcement


minimum steel area = ( 0.12 / 100 ) x 1000 x 110
= 132
mm2
For Distribution steel , mild steel Fe250 reinforcement
minimum steel area = ( 0.15 / 100 ) x 1000 x 110
= 165
mm2
Use 6 mm O
spacing of bar = Area of one bar x 1000 / required area in m 2 / m
= 28.26 x 1000 /165
= 171
Use 6 mm O @ 160 mm c/c = 177 mm2.
Note that the positive bars cannot be curtailed as the remaining bars in the internal
spans ( + ve moment ) will not provide minimum area.
Provide 50 % Ast at end support top bars i.e. 292 / 2 = 146 mm 2 .
Use 8 mm O
spacing of bar = Area of one bar x 1000 / required area in m 2 / m
= 50.24 x 1000 /146
= 344
Use 8 mm O @ 340 mm c/c = 148 mm2.

Check for shear : Maximum shear = 18.9 KN


Actual Shear stress = Vu / b x d
= 18.9 x 103 / 1000 x 90
= 0.210

N / mm2

< (

) N / mm2 ( too small )

For bars at support


d = 90

mm

As = 357
mm2 .
100 x As / b x d = 100 x 357 / 1000 x 90
= 0.397
Design shear strength c = 0.85 0.8 x fck ( 1 + 5 x - 1 )
6x
= 0.8 x fck / 6.89 Pt , but not less than 1.0
= 0.8 x 15 / 6.89 x 0.397
= 4.4
Design shear strength c = 0.85 0.8 x 15 ( 1 + 5 x 4.4 - 1 )
6 x 4.4
= 0.42
IS 456-2000 Table 19 from table 7-1
for Pt = 0.397
c = 0.42
N / mm2
IS 456-2000 clause 40.2.1.1
0.25 difference 0.11
k = 1.3
for 110 mm slab depth
0.103 difference
?
0.04532
Design shear strength = 1.3 x 0.42
= 0.546
N / mm2 .( O.K.)
At point of contraflexure i.e. 0.15 x l from B
with positive moment reinforcement ( 292 mm2 )
Vu = 18.9 - 0.15 x 3 x 10.5
= 14.18
KN
Actual Shear stress = Vu / b x d
= 14.18 x 103 / 1000 x 90
= 0.158

N / mm2

< (

) N / mm2 ( too small )

For bars at support


d = 90

mm

As = 292
mm2 .
100 x As / b x d = 100 x 292 / 1000 x 90
= 0.324
Design shear strength c = 0.85 0.8 x fck ( 1 + 5 x - 1 )
6x
= 0.8 x fck / 6.89 Pt , but not less than 1.0

= 0.8 x 15 / 6.89 x 0.324


= 5.4
Design shear strength c = 0.85 0.8 x 15 ( 1 + 5 x 5.4 - 1 )
6 x 5.4
OR
= 0.39
IS 456-2000 Table 19 from table 7-1
for Pt = 0.324
c = 0.38
N / mm2
IS 456-2000 clause 40.2.1.1
0.25 difference 0.11
k = 1.3
for 110 mm slab depth
0.176 difference
?
Design shear strength = 1.3 x 0.38
= 0.494
N / mm2 .( O.K.)
Check for development length : Span AB is critical for checking this requirement
At support A
Vu = 0.4 x w x l + 0.45 x w x l
= 0.4 x 6 x 3 + 0.45 x 4.5 x 3 = 13.28
KN
Pt = 100 x As / b x d
At A ,
= 100 x 292 / 1000 x 90
= 0.324
From equation
OR
Pt = 50 1-1-(4.6 / fck) x (Mu / b x d2)

Mu1 = 0.87 x fy x Ast x d ( 1 - fy

fy / fck

= 0.87 x 415 x 292 x 90 (

Mu1 / b x d = 1.064
Mu1 = 1.064 x 1000 x 902 x 10-6
= 8.62
KNm
Assuming L0 = 8 O
(HYSD Fe415 steel )

we get ,

Development length of bars Ld =

O s / 4 x bd

0.07744

= O x 0.67 x 415 / 4 x 1

= 9.48839
= 8.64

=56.4 O

1.3 x ( Mu1 / Vu ) + L0 Ld
1.3 x ( 8.64 x 106 / 13.28 x 103 ) + 8 O 56.4 O
845.7831 + 8 O
56.4 O
48.4 O 845.783
which gives
O 17.47
mm
.( O.K.)
At support B, point of contraflexure is assumed at 0.15 x l from B
Vu = 18.9 - 0.15 x 3 x 10.5
= 14.18
KN
Mu1 = 8.64
KNm
as before
L0 =

12 O ( actual anchorage is more than 12 O but L 0 is limited to 12 O or d ,


i.e. 90 mm whichever is greater )
1.3 x ( Mu1 / Vu ) + L0 Ld
1.3 x ( 8.64 x 106 / 14.18 x 103 ) + 12 O 56.4 O
792.1016 + 12 O 56.4 O

44.4 O 792.102
which gives
O 17.84
mm
.( O.K.)
Check for deflection : Maximum positive moment occurs in span AB. Therefore, this check is critical in span AB
Basic ( span / d ) ratio = 26
Pt = 100 x Ast / b x d = 100 x 292 / 1000 x 90
= 0.324
IS 456-2000 clause 23.2.1 fig-4 ,for tension reinforcement
modification factor = 1.34
( span / d ) ratio permissible = 1.34 x 26
= 34.84
Actual (span / d ) ratio = 3000 / 90
= 33.33
<
34.84
.( O.K.)
For span BC :
Basic ( span / d ) ratio = 26
Pt = 100 x Ast / b x d = 100 x 228 / 1000 x 90
= 0.253
IS 456-2000 clause 23.2.1 fig-4 ,for tension reinforcement
modification factor = 1.6
( span / d ) ratio permissible = 1.6 x 26
= 41.6
Actual (span / d ) ratio = 3000 / 90
= 33.33
<
41.6
.( O.K.)
Check for cracking : IS 456-2000 , clause 26.3.3
(1) Main bars : maximum spacing permitted = 3 x effective depth of slab or 300 mm whichever is sm
= 3 x 90
= 270
mm
or 300 mm i.e. 270 mm
spacing provided = 220
mm
< 270 mm
.( O.K.)
(2 )Distribution bars : maximum spacing permitted = 5 x effective depth of slab or 450 mm which
= 5 x 90
= 450 mm
spacing provided = 160
mm
< 450 mm
.( O.K.)

IS 456-2000 Clause -22.5


( 22.5.1 ) Unless more exact estimates are made, for
beams of uniform cross-section which support
substantially uniformly distributed loads over three or
more spans which do not differ by more than 15 percent
of the longest, the bending moments and shear forces
used in design may be obtained using the coefficients
given in Table 12 and Table 13 respectively.
For moments at supports where two unequal spans
meet or in case where the spans are not equally loaded,
the average of the two values for the negative moment
at the support may be taken for design.
Where coefficients given in Table 12 are used for
calculation of bending moments, redistribution referred
to in 22.7 shall not be permitted.
(22.5.2 ) Beams and Slabs Over Free End Supports
Where a member is built into a masonry wall which
develops only partial restraint, the member shall be
designed to resist a negative moment at the face of the
support of Wl / 24 where W is the total design load
and I is the effective span, or such other restraining
moment as may be shown to be applicable. For such a
condition shear coefficient given in Table 13 at the
end support may be increased by 0.05.
Table 12 Bending Moment coefficients
Span moments
Support moments
Type of load
Dead load and imposed
load ( fixed )
imposed load
( not fixed )

Near middle At middle of At support next to


of end span interior span the end support
+ 1 / 12

+ 1 / 16

-1 / 10

+ 1 / 10

+ 1 / 12

-1 / 9

NOTE -For obtaining the bending moment, the coefficient shall be multiplied by the total
design load and effective span.
Table 13 Shear Force coefficients
Type of load
Dead load and imposed
load ( fixed )
imposed load
( not fixed )

At end
support

At support next to the end support

Outer side

Inner side

0.4

0.6

0.55

0.45

0.6

0.6

( 0.15 l1 )

( 0.3 l1 )
900

450
8 O @ 340 c/c
110

( 0.3 l1 )
900

( 0.3 l1 )
900
10 O @ 220 c/c

90
6 O @ 160 c/c
8 O @ 440 c/c
+ 10 O @ 440 c/c
3000

8 O @ 220 c/c

3000

cement ( 292 mm2 )

0.87 x fy x Ast x d ( 1 - fy x Ast / b x d x fck )


0.87 x 415 x 292 x 90 ( 1 - 415 x 292 / 1000 x 90 x 15 ) x 10 -6
( 1 - 0.0898 )
KNm

in span AB

r 300 mm whichever is small


or 300 mm i.e. 270 mm
.( O.K.)
of slab or 450 mm whichever is small
or 450 mm i.e. 450 mm
.( O.K.)

ts
Support moments
At other interior
supports

-1 / 12
-1 / 9

e multiplied by the total

At all other
interior
supports
0.5
0.6

( 0.3 l1 )
900
10 O @ 220 c/c

3000

Design of Simply supported two way slab


residential building drawing room 4.3 m x 6.55 m
It is supported on 350 mm thick walls on all four sides.
material M15 grade concrete
HYSD reinforcement of grade Fe415

given

Solution :
Consider 1 m wide strip. Assume 180 mm thick slab.
lx = 4.3 + 0.18 = 4.48 say 4.5 m
ly = 6.55+0.18 = 6.73 say 6.75 m
Dead load : self 0.18 x 25 = 4.5

KN / m2

floor finish = 1.0

KN / m2

Live load ( residence ) = 2.0

KN / m2

Total

7.5

KN / m2

For 1 m wide strip


Pu = 1.5 x 7.5
= 11.25
KN / m
ly / lx = 6.75 / 4.5
= 1.5
IS 456-2000 Table -27
Mux =
x x w x lx2 =
Muy =

y x w x lx2 =

From Table 6-3

0.104 x 11.25 x 4.52 =

23.7

KNm

0.046 x 11.25 x 4.52 =


Q = 2.07

10.48

KNm

M / Q x b
= 23.7 x 10

drequired =

/ 2.07 x 1000

= 107
mm,
dshort provided = 180 - 15 - 5 ( assume 10 O bar )
= 160
mm,
> 107 mm
dlong provided = 160 - 10
= 150
mm,
> 107 mm
Larger depth is provided due to deflection check.
Mu / b x d2 ( short ) =

23.7 x 10 6 / 1000 x (160)2


= 0.926

Pt = 50 1-1-(4.6 / fck) x (Mu / b x d2)


fy / fck

.( O.K.)

= 50 1-1-(4.6 / 15) x (0.926)


415 / 15
= 50 [(1-0.846) x 15 / 415 ]
= 0.28%
Ast ( short ) = 0.28 x 1000 x 160 / 100
= 448
mm2
Provide 10 mm O bar
spacing of bar = Area of one bar x 1000 / required area in m 2 / m
= 78.5 x 1000 / 448
= 175.223
mm
Provide 10 mm O bar@170 mm c/c = 462

mm2.

( short span )

Mu / b x d ( long ) =
2

10.48 x 10 6 / 1000 x (150)2


= 0.466

Pt = 50 1-1-(4.6 / fck) x (Mu / b x d2)


fy / fck
= 50 1-1-(4.6 / 15) x (0.466
415 / 15
= 50 [(1-0.926) x 15 / 415 ]
= 0.134%
Ast ( long ) = 0.134 x 1000 x 150 / 100
= 201
mm2
For HYSD Fe415 minimum reinforcement 0.12 %
Minimum steel = ( 0.12 /100 ) x 1000 x 180
= 216
mm2.
Provide 8 mm O bar
spacing of bar = Area of one bar x 1000 / required area in m 2 / m
= 50.24 x 1000 / 216
= 232.593
mm
Provide 8 mm O bar@ 230 mm c/c = 218
mm2.
( long span )
The bars cannot be bent or curtailed because if 50 % of long span bars are curtailed ,
the remaining bars will be less than minimum
At top on support , provide 50 % of bars of respective span to take into account negative
moment due to slab nature.

Check for development length : Vu = 11.25 x ( 4.5 / 2 )


Long span

= 25.31 KN

Assuming L0 = 8 O
(HYSD Fe415 steel )
Pt = 100 x As / b x d
= 100 x 218 / 1000 x 150
= 0.145
From equation

OR

Pt = 50 1-1-(4.6 / fck) x (Mu / b x d2)

Mu1 = 0.87 x fy x Ast x d ( 1 - fy x

fy / fck

= 0.87 x 415 x 218 x 150 (

Mu1 / b x d2 = 0.5023
Mu1 = 0.5023 x 1000 x 1502 x 10-6
= 11.30
KNm
Development length of bars Ld = O s / 4 x bd
=56 O (From Table 7-6 )
1.3 x ( Mu1 / Vu ) + L0 Ld
we get ,

= 11.8063
= 11.3313

1.3 x ( 11.30 x 106 / 25.31 x 103 ) + 8 O 56 O


580.403 + 8 O
56 O
48 O 580.403
which gives
O 12.09
mm
.( O.K.)
Vu = 11.25 x ( 4.5 / 2 ) = 25.31 KN
Short span
Assuming L0 = 8 O
(HYSD Fe415 steel )
Pt = 100 x As / b x d
= 100 x 462 / 1000 x 160
= 0.289
From equation
Pt = 50 1-1-(4.6 / fck) x (Mu / b x d2)

OR
Mu1 = 0.87 x fy x Ast x d ( 1 - fy x

fy / fck
Mu1 / b x d = 0.9595
Mu1 = 0.9595 x 1000 x 1602 x 10-6
= 24.56
KNm
Development length of bars Ld = O s / 4 x bd
=56 O (From Table 7-6 )
1.3 x ( Mu1 / Vu ) + L0 Ld
we get ,

= 0.87 x 415 x 462 x 160 (


= 26.6888
= 24.5539

1.3 x ( 24.56 x 106 / 25.31 x 103 ) + 8 O 56 O


1261.48 + 8 O
56 O
48 O 1261.48
which gives
O 26.28
mm
.( O.K.)
Note that the bond is usually critical along long direction.

Check for shear : This is critical along long span


Vu = 25.31
KN
Shear stress = Vu / b x d
= 25.31 x 103 / 1000 x 150
2
= 0.169
N / mm2 < ( C ) N / mm
( too small )
100 x As / b x d = 100 x 218 / 1000 x 150
= 0.145
from table 7-1
for Pt = 0.145
c = 0.28 N / mm2
-0.05
25 difference
IS 456-2000 clause 40.2.1.1
?
20 difference
k = 1.24
for 180 mm slab depth
Design shear strength = 1.24 x 0.28
= 0.347
N / mm2 .( O.K.)
OR
Design shear strength c = 0.85 0.8 x fck ( 1 + 5 x - 1 )
6x
= 0.8 x fck / 6.89 Pt , but not less than 1.0
= 12.0114
Design shear strength c = 0.85 0.8 x 15 ( 1 + 5 x 12.01 - 1 )
6 x 12.011
=
0.278
IS 456-2000 clause 40.2.1.1
-0.05
25 difference
k = 1.24
for 180 mm slab depth
?
20 difference
Design shear strength = 1.24 x 0.278
= 0.345
N / mm2 .( O.K.)
Check for deflection : This check shall be done along short span
Basic ( span / d ) ratio = 20
Pt = 100 x Ast / b x d = 100 x 462 / 1000 x 160
= 0.289
IS 456-2000 clause 23.2.1 fig-4 ,for tension reinforcement
modification factor = 1.44
( span / d ) ratio permissible = 1.44 x 20
= 28.8
Actual (span / d ) ratio = 4480 / 160
= 28.00
<
28.8
.( O.K.)
Check for cracking : IS 456-2000 , clause 26.3.3

-0.04

-0.04

(1) Main bars : maximum spacing permitted for short span steel =
=
spacing provided = 170
mm
< 300 mm
(2) Distribu. bars : maximum spacing permitted for long span steel =
=
spacing provided = 230
mm
< 450 mm

3 x effective depth of slab or 300 m


3 x 160 = 480
.( O.K.)
5 x effective depth of slab or 450 m
5 x 150 = 750
.( O.K.)

Table - 26 Bending moment coefficients for rectangular panels supported on four sides with provis

short span coefficient


( Values of l

Case No. Type of Panel and Moments considered


1.0

1.1

1.2

1.3

0.032
0.024

0.037
0.028

0.043
0.032

0.047
0.036

Negative moment at continuous edge

0.037

0.043

0.048

0.051

Positive moment at mid-span

0.028

0.032

0.036

0.039

Negative moment at continuous edge

0.037

0.044

0.052

0.057

Positive moment at mid-span


Two Adjacent Edges Discontinuous:

0.028

0.033

0.039

0.044

Negative moment at continuous edge


Positive moment at mid-span

0.047
0.035

0.053
0.040

0.060
0.045

0.065
0.049

Two Short Edges Discontinuous:


Negative moment at continuous edge
Positive moment at mid-span

0.045
0.035

0.049
0.037

0.052
0.040

0.056
0.043

0.035

0.043

0.051

0.057

0.057

0.064

0.071

0.076

0.043

0.048

0.053

0.057

(One Short Edge Continuous) :


Negative moment at continuous edge
Positive moment at mid-span

0.043

0.051

0.059

0.065

Four-Edges Discontinuous:
Positive moment at mid-span

0.056

0.064

0.072

0.079

Interior Panels:
Negative moment at continuous edge
Positive moment at mid-span

One Short Edge Discontinuous:

One long Edge Discontinuous:

Two Long Edges Discontinuous:


Negative moment at continuous edge
Positive moment at mid-span

Three Edges Discontinuous


(One Long Edge Continuous):
Negative moment at continuous edge

Positive moment at mid-span


Three Edges Discrmntinuous

Table - 27 Bending moment coefficients for slabs spanning in two directions at right angles , simp

ly / lx

1.0

1.1

1.2

1.3

1.4

0.062

0.074

0.084

0.093

0.099

0.062

0.061

0.059

0.055

0.051

0.87 x fy x Ast x d ( 1 - fy x Ast / b x d x fck )


0.87 x 415 x 218 x 150 ( 1 - 415 x 218 / 1000 x 150 x 15 ) x 10

-6

( 1 - 0.0402 )
KNm

0.87 x fy x Ast x d ( 1 - fy x Ast / b x d x fck )


0.87 x 415 x 462 x 160 ( 1 - 415 x 462 / 1000 x 160 x 15 ) x 10

-6

( 1 - 0.0799 )
KNm

A
350
350

6550

350

460

10 O @ 340 c/c
10 O @ 170 c/c

690

4300

8 O @ 230 c/c

690

4650

8 O @ 460 c/c

460
350
6900
A

PLAN
46010 O @ 340 c/c

460

180
v
v
v
v
10 O @ 170
v
v
v c/c
v
8 O @ 230 c/c
350

4300

Section A-A

v
v

v
v

350

tive depth of slab or 300 mm whichever is small


mm
or 300 mm i.e. 300 mm
.( O.K.)
tive depth of slab or 450 mm whichever is small
mm
or 450 mm i.e. 450 mm
.( O.K.)

four sides with provision for torsion at corners

ort span coefficient x


( Values of ly / lx )

Long span
coefficient y for all
values of ly / lx

1.4

1.5

1.75

2.0

0.051
0.039

0.053
0.041

0.060
0.045

0.065
0.049

0.032
0.024

0.055

0.057

0.064

0.068

0.037

0.041

0.044

0.048

0.052

0.028

0.063

0.067

0.077

0.085

0.037

0.047

0.051

0.059

0.065

0.028

0.071
0.053

0.075
0.056

0.084
0.063

0.091
0.069

0.047
0.035

0.059
0.044

0.060
0.045

0.065
0.049

0.069
0.052

0.035

0.063

0.068

0.080

0.088

0.045
0.035

0.080

0.084

0.091

0.097

0.060

0.064

0.069

0.073

0.043

0.071

0.076

0.087

0.096

0.057
0.043

0.085

0.089

0.100

0.107

0.056

at right angles , simply supported on Four sides

1.5

1.75

2.0

2.5

3.0

0.104

0.113

0.118

0.122

0.124

0.046

0.037

0.029

0.020

0.014

Design of Continuous Beam


An R.C.C. floor is used as a banking hall
Design the beams B10-B11-B12.
The Slab thickness is 120 mm .
Live load = 3 KN / m2
6m
2
(Given )
Floor finish = 1 KN / m
Rib size = 230 mm x 450 mm
Column = 300 mm x 300 mm
Main beams = 300 mm x 570 mm overall .
Material M15 grade concrete
HYSD reinforcement of grade Fe415 .
Solution : ( a ) Load calculations and analysis :
Slab 120 mm thick 0.12 x 25 = 3 + 0 KN / m 2
Floor finish = 1 + 0 KN /m2
Live load = 0 + 3 KN / m2

B10

6m

Total 4 + 3 KN / m2
Load on beam = 3 ( 4 + 3 ) = 12 + 9 KN / m
Self wt. = 0.23 x 0.45 x 25 = 2.58 + 0 KN / m
Total 14.58 + 9 KN / m
Factored Load = 1.5 ( 14.58 + 9 )
= 21.87 + 13.5
say ( 22 + 14 ) KN /m .
Case ( a ) Maximum moment at B
36 KN / m

6m

22 KN / m

36 KN / m

6m

6m

Using three moment equation for span ABC


MA ( L1 / I1 ) + 2MB ( L1 / I1 + L2 / I2 ) + MC ( L2 / I2 ) = - 6 A1a1 / ( I1 L1 ) -6 A2a2 / ( I2L2)
36 KN / m

36 KN / m

162

C
162

A1 = ( 2 / 3 ) x Base x h1
A2 = ( 2 / 3 ) x Base x h1
= ( 2 / 3 ) x 6 x 162
= ( 2 / 3 ) x 6 x 162
= 648
= 648
a1 = 3 m
a2 = 3 m
MA ( 6 / I ) + 2MB ( 6 / I + 6 / I ) + MC ( 6 / I ) = - [ 6 x 648 x 3 / ( I x 6 ) ] - [ 6 x 648 x 3 / ( I x 6) ]
6MA + 24MB + 6MC = - [ 1944 ] - [ 1944 ]

4MB + MC = - 648
As MA = 0
.( 1 )
Using three moment equation for span BCD
MB ( L2 / I2 ) + 2MC ( L2 / I2 + L3 / I3 ) + MD ( L3 / I3 ) = - 6 A2a2 / ( I2 L2 ) -6 A3a3 / ( I3L3)
36 KN / m

22 KN / m

162

99

A2 = ( 2 / 3 ) x Base x h2
A3 = ( 2 / 3 ) x Base x h3
= ( 2 / 3 ) x 6 x 162
= ( 2 / 3 ) x 6 x 99
= 648
= 396
a2 = 3 m
a3 = 3 m
MB ( 6 / I ) + 2MC ( 6 / I + 6 / I ) + MD ( 6 / I ) = - [ 6 x 648 x 3 / ( I x 6 ) ] - [ 6 x 396 x 3 / ( I x 6) ]
6MB + 24MC + 6MD = - [ 1944 ] - [ 1188 ]
MB + 4MC = - 522

As MD = 0

4MB + MC = - 648

.( 1 )

.( 2 )

MB + 4MC = - 522

.( 2 )
By putting Value of MB from Equation ( 2 ) into Equation ( 1 )
4(- 522 - 4MC ) + MC = - 648
- 2088 - 16 MC + MC = - 648
15 MC = - 1440
MC = - 96 KNm
By putting Value of MC into Equation ( 1 )
4MB + ( - 96 ) = - 648
4 MB = - 552
MB = - 138 KNm
36 KN / m

6m
162
(+)

22 KN / m

36 KN / m

B
138
(-)

6m

(-)

(-)

99
(+)

51

45
(+)

(+)

6m

96
162

93

(+)
(-)

6 m Distance
3 m Distance
6 m Distance
3 m Distance
6 m Distance

138 KNm
(?)
96 KNm
(?)
42 KNm

69
48

3 m Distance

96

(?)

21

138
85

MB = - 138 KNm

115
82

2.36 m
50
131

MC = - 96 KNm

101

VA x 6 -36 x 6 x 3 = -138

X dist.

VA = 85 KN

6 - X dist.

VA + VB = 36 x 6

131 X =85 ( 6 - X )

VB = 216 - 85

216 X = 510

VB = 131 KN

X = 2.36 m

MB = - 138 KNm

VD x 6 -22 x 6 x 3 = -96

50 x 12 + VC x 6 - 36 x 6 x 3 - 22 x 6 x 9 = -138

VD = 50 KN

Vc = 183 KN

VC + VD = 22 x 6

VC + 82 = 183

VC = 132 - 50

VC = 101 KN

VC = 82 KN

MC = - 96 KNm
85 x 12 + VB x 6 - 36 x 6 x 9 - 36 x 6 x 3 = - 96
VB = 246 KN
VB + 131 = 246

VB = 115 KN
Three cases are considered for getting maximum values of moments. Case ( a ) gives maximum
negative moment at B.The same moment shall be used at C also because of symmetry. Case ( b )
gives the maximum positive moment in span BC while the case ( c ) gives maximum positive
moments in span AB and CD.
Factored maximum moments :
B or C , negative moment = 138 KNm
AB ( + ) = CD ( + ) = 110 KNm
BC ( + ) = 58 KNm
BC ( minimum - ve ) = 5 KNm
The moment redistribution shall be now carried out. Maximum negative moment = 138 KNm.
reduce it by 20 % . Then Mu = 0.8 x 138 = 110.4 KNm. For all the cases , redistribute ( increase or
decrease ) the moment at B or C = 110.4 KNm ( Hogging ).
Maximum design moments :
Support B or C = 110.4 KNm > 0.7 x 138 KNm ( O.K.)
( As per IS 456-2000
AB or CD ( + ) = 111.6 KNm > 0.7 x 110 KNm
BC ( - ) = 11.6 KNm
BC ( + ) = 51.6 KNm > 0.7 x 58 KNm.
Note that after redistribution , the design positive moments also have been reduced.
( b ) Design for flexure :
Span AB or CD
Mu ( + ) = 111.6 KNm.
The beam acts as a flanged beam

For T-beams , bf = ( l0 / 6 ) + bw + 6 Df
bf = ( 0.7 x 6000 / 6 ) + 230 + 6 x 120
( As per IS456-2000 ,Clause 23.1.2 , Note )
= 1650 mm > 3000 mm
Assuming one layer of 20 mm diameter bars
d = 450 + 120 -25 - 10 = 535 mm .
Minimum Ast = ( 0.205 / 100 ) x 230 x 535 = 252 mm 2
( As per IS456-2000 ,Clause 26.5(
bf / bw = 1650 / 230 = 7.17
Df / d = 120 / 535 = 0.224

For bf /bw = 7

0.01 diff.

Mu,lim / fck bw d = 0.671


2

( As per SP:16 ,Table 58 ) 0.006 diff.


Mu.lim = 0.671 x 15 x 230 x 5352 x 10-6
= 662.597 KNm > 111.6 KNm

If Mu < Mu,lim :

For bf /bw = 8

0.014

0.016

For 0.224

-0.0084 -0.0096
0.6566

0.743

1 diff
0.0864
0.83 diff
?
0.07171
design as under-reinforced section (singly reinforced beam) as explained below.

Ast = Mu / 0.87 x fy x lever arm


where lever arm = d - Df / 2 = 535 - 120 / 2 = 535 - 60
Ast = 111.6 x 106 / 0.87 x 415 x ( 535 - 60 )
= 651

mm2 .

Provide 6 - 12 mm O = 678 mm2 .


Span BC :
Mu ( + ) = 51.6 KNm
Ast = 51.6 x 106 / 0.87 x 415 x ( 535 - 60 )
= 301
mm2 .
Provide 3 - 12 mm O = 339 mm2 .
In span AB , curtail 3 - 12 O at 300 mm ( 0.05 ) from A and at 900 mm ( 0.15 ) from B.
Continue 3 - 12 O in span BC as required for flexure.
Support B or C :
Mu ( - ) = 110.4 KNm
Mu / bd2 = 110.4 x 106 / 230 x 5352 = 1.68 < 2.07.
The section is under-reinforced.
Pt = 50

( From Table 6-3 )

1 - 1 - ( 4.6 / fck) x ( Mu / bd2 )


fy / fck

Pt = 50 1 - 1 - ( 4.6 / 15 ) x ( 1.68 )
415 / 15
= 0.549
Ast = ( 0.549 / 100 ) x 230 x 535 = 676 mm 2
20 % of steel should be carried through the span = 0.2 x 676 = 135 mm 2

603

135

Provide 2- 10 mm O anchor bars = 157 mm 2 . At support , provide 3 - 16 mm O extra at top = 603 mm 2


one of which may be curtailed at 0.15 = 900 mm from centre of support B and remaining 2 - 16 mm O
at 0.25 = 1500 mm from B .

3m
B11

B12
3m

6m

6m

Case ( b ) Maximum positive moment in span BC


22 KN / m

6m

22 KN / m

36 KN / m

6m

6m

Using three moment equation for span ABC


MA ( L1 / I1 ) + 2MB ( L1 / I1 + L2 / I2 ) + MC ( L2 / I2 ) = - 6 A
22 KN / m

36 KN / m

B
162

99

A1 = ( 2 / 3 ) x Base x h1
= ( 2 / 3 ) x 6 x 99
= 396
a1 = 3 m
x 3 / ( I x 6) ]

A2 = ( 2 / 3 ) x Base x h
= ( 2 / 3 ) x 6 x 162
= 648
a2 = 3 m

MA ( 6 / I ) + 2MB ( 6 / I + 6 / I ) + MC ( 6 / I ) = - [ 6 x 396 x 3 /
6MA + 24MB + 6MC = - [ 1188 ] - [ 1944 ]

4MB + MC = - 522
As MA = 0
Using three moment equation for span BCD
MB ( L2 / I2 ) + 2MC ( L2 / I2 + L3 / I3 ) + MD ( L3 / I3 ) = - 6 A
36 KN / m

22 KN / m

162

99

A2 = ( 2 / 3 ) x Base x h2
A3 = ( 2 / 3 ) x Base x h
= ( 2 / 3 ) x 6 x 162
= ( 2 / 3 ) x 6 x 99
= 648
= 396
a2 = 3 m
a3 = 3 m
x 3 / ( I x 6) ]

MB ( 6 / I ) + 2MC ( 6 / I + 6 / I ) + MD ( 6 / I ) = - [ 6 x 648 x 3 /
6MB + 24MC + 6MD = - [ 1944 ] - [ 1188 ]
MB + 4MC = - 522

As MD = 0

4MB + MC = - 522

.( 1 )

MB + 4MC = - 522

.( 2 )
By putting Value of MB from Equation ( 2 ) into Equation ( 1 )
4(- 522 - 4MC ) + MC = - 522
- 2088 - 16 MC + MC = - 522
15 MC = - 1566
MC = - 104 KNm
By putting Value of MC into Equation ( 1 )
4MB + ( - 104 ) = - 522
4 MB = - 418
MB = - 104 KNm
22 KN / m

6m

A
99

B
104
(-)

6m

162

( 162 - 69 = 93 KNm )
( 99 - 48 = 51 KNm )

22 KN / m

36 KN / m

C104
(-)

6m
99
(+)

47

58

47

(+)

(+)

(+)

(-)

(-)

( 162 - ( 96 + 21 ) = 45 KNm )
104

85

104

108

48.67

83.33

131
131 X =85 ( 6 - X )

2.21 m
48.67

216 X = 510
X = 2.36 m

83.33

MC = - 104 KNm

108

MB = - 104 KNm

VD x 6 -22 x 6 x 3 = -104

48.67 x 12 + VC x 6 - 36 x 6 x 3

VD = 48.67 KN

Vc = 191.33 KN

VC + VD = 22 x 6

VC + 83.33 = 191.33

VC = 132 - 48.67

VC = 108 KN

VC = 83.33 KN

MC = - 104 KNm

48.67 x 12 + VB x 6 - 36 x 6 x 3 VB = 191.33 KN
VB + 83.33 = 191.33
VB = 108 KN

gives maximum
mmetry. Case ( b )
mum positive

= 138 KNm.
ibute ( increase or

( As per IS 456-2000 , Clause 37.1.1 )

IS 456-2000 ,Clause 37.1.1 Redistribution of mom


Continuous Beams and Frames ( b ) The ultimate moment of resistance provided at
member is not less than 70 percent of the moment
obtained from an elastic maximum moment diagram
appropriate combinations of loads.

( c ) The elastic moment at any section in a me


particular combination of loads shall not be reduce
30 % of the numerically largest moment given an
elastic maximum moments diagram for the partic
covering all appropriate combination of loads.

1.2 , Note )

6-2000 ,Clause 26.5( a ) )


For bf /bw = 8

0.671
explained below.

IS 456-2000 ,Clause 23.1.2 Effective width of flan


In the absence of more accurate determination , the
width of flange may be taken as the following but in
greater than the breadth of the web plus half the su
distances to the adjacent beams on either side :
( a ) For T-beams , bf = ( l0 / 6 ) + bw + 6 Df
( b ) For L-beams , bf = ( l0 / 12 ) + bw + 3 D
Where ,
bf = effective width of flange ,

l0 = distance between points of zero moments in the


bw = breadth of the web ,
Df = thickness of flange , and
b = actual width of the flange.

NOTE - For continuous beams and frames , ' l0 ' ma


as 0.7 times the effective span.

IS 456-2000 Clause 26.5 Requirements of Reinfo


Structural Members
26.5.1 Beams
26.5.1.1 Tension Reinforcement

a ) Minimum reinforcement - The minimum area o


shall not be less than that given by the following :
As / b d = 0.85 / fy

where ,
As = minimum area of tension reinforcement ,
b = breadth of the beam or the breadth of the web o
d = effective depth , and
fy = characteristic strength of reinforcement in N / m

b ) Maximum reinforcement - The maximum area


reinforcement shall not exceed 0.04 b D.
Minimum steel %
For mild steel
100 As / b d = 100 x 0.85 / 250 = 0.34
For HYSD steel , Fe415 grade
100 As / b d = 100 x 0.85 / 415 = 0.205
For HYSD steel , Fe500 grade
100 As / b d = 100 x 0.85 / 500 = 0.17

extra at top = 603 mm 2 ,


remaining 2 - 16 mm O

Table 6-3
Limiting Moment of resistance factor Q lim, N / mm
For singly reinforced rectangular sections

fck
N
2
/ mm

15
20
25
30

fy, N / mm2
250
2.22
2.96
3.70
4.44

415
2.07
2.76
3.45
4.14

500
2.00
2.66
3.33
3.99

t in span BC

Case ( c ) Maximum positive mome


36 KN / m

6m

MC ( L2 / I2 ) = - 6 A1a1 / ( I1 L1 ) -6 A2a2 / ( I2L2)

Using three moment equation for s


MA ( L1 / I1 ) + 2MB ( L

36 KN / m

36 KN / m

A
162

/ 3 ) x Base x h1
( 2 / 3 ) x 6 x 162

A1 = ( 2 / 3 ) x Base x h
= ( 2 / 3 ) x 6 x 162
= 648
a1 = 3 m

6 / I ) = - [ 6 x 396 x 3 / ( I x 6 ) ] - [ 6 x 648 x 3 / ( I x 6) ]

MA ( 6 / I ) + 2MB ( 6 / I + 6 / I ) + M

6MA + 24MB + 6MC = - [ 1944 ] - [ 1

.( 1 )

MD ( L3 / I3 ) = - 6 A2a2 / ( I2 L2 ) -6 A3a3 / ( I3L3)

4MB + MC = - 522
Using three moment equation for s
MB ( L2 / I2 ) + 2MC ( L

22 KN / m

22 KN / m

B
99

/ 3 ) x Base x h3
( 2 / 3 ) x 6 x 99

A2 = ( 2 / 3 ) x Base x h
= ( 2 / 3 ) x 6 x 99
= 396
a2 = 3 m

6 / I ) = - [ 6 x 648 x 3 / ( I x 6 ) ] - [ 6 x 396 x 3 / ( I x 6) ]

MB ( 6 / I ) + 2MC ( 6 / I + 6 / I ) + M

88 ]

6MB + 24MC + 6MD = - [ 1188 ] - [ 1


.( 2 )

MB + 4MC = - 522

.( 1 )

4MB + MC = - 522

.( 2 )
( 2 ) into Equation ( 1 )

MB + 4MC = - 522
By putting Value of M
4(- 522 - 4MC ) + M
- 2088 - 16 MC + M
15 MC = - 1566
MC = - 104.4 KNm
By putting Value of M
4MB + ( - 104.4 ) = - 522
4 MB = - 417.6
MB = - 104.4 KNm
36 KN / m

6m
162
(+)

6 m Distance
104 KNm
3 m Distance
(?)
52
( 99 - 52 = 47 KNm )
Moment in span BC = ( 162 - 104 ) = 58 KNm

B
104
(-)

110
(+)
(-)

104

MB = - 104 KNm
VA x 6 -22 x 6 x 3 = -104

X dist.

48.67

VA = 48.67 KN

6 - X dist.

83.33

VA + VB = 22 x 6

83.33 X = 48.67 ( 6 - X )

VB = 132 - 48.67

132 X = 292.02

VB = 83.33 KN

X = 2.21 m

12 + VC x 6 - 36 x 6 x 3 - 22 x 6 x 9 = -104

33 = 191.33

12 + VB x 6 - 36 x 6 x 3 - 22 x 6 x 9 = - 104

33 = 191.33

1 Redistribution of moments in
rames of resistance provided at any section of a
percent of the moment at that section
aximum moment diagram covering all

at any section in a member due to a


ads shall not be reduced by more than
rgest moment given anywhere by the
diagram for the particular member ,
mbination of loads.

90.67
66

2.52 m

125.33

MC = - 104 KNm
VD x 6 -36 x 6 x 3 = -104
VD = 90.67 KN
VC + VD = 36 x 6
VC = 216 - 90.67
VC = 125.33 KN

2 Effective width of flange urate determination , the effective


n as the following but in no case
the web plus half the sum of the clear
eams on either side :
6 ) + bw + 6 Df
12 ) + bw + 3 Df

s of zero moments in the beam ,

ms and frames , ' l0 ' may be assumed


pan.

equirements of Reinforcement for

nt - The minimum area of tension reinforcement

iven by the following :

on reinforcement ,
the breadth of the web of T- beam ,

of reinforcement in N / mm 2 .

nt - The maximum area of tension


eed 0.04 b D.

50 = 0.34

15 = 0.205

00 = 0.17

tor Q lim, N / mm2

sections

/ mm2
550
1.94
2.58
3.23
3.87

) Maximum positive moment in span AB and CD


36 KN / m

22 KN / m

6m

6m

ree moment equation for span ABC


I1 ) + 2MB ( L1 / I1 + L2 / I2 ) + MC ( L2 / I2 ) = - 6 A1a1 / ( I1 L1 ) -6 A2a2 / ( I2L2)
36 KN / m

22 KN / m

C
99

/ 3 ) x Base x h1
A2 = ( 2 / 3 ) x Base x h1
( 2 / 3 ) x 6 x 162
= ( 2 / 3 ) x 6 x 99
= 396
a2 = 3 m
) + 2MB ( 6 / I + 6 / I ) + MC ( 6 / I ) = - [ 6 x 648 x 3 / ( I x 6 ) ] - [ 6 x 396 x 3 / ( I x 6) ]

4MB + 6MC = - [ 1944 ] - [ 1188 ]

= - 522
As MA = 0
.( 1 )
ree moment equation for span BCD
I2 ) + 2MC ( L2 / I2 + L3 / I3 ) + MD ( L3 / I3 ) = - 6 A2a2 / ( I2 L2 ) -6 A3a3 / ( I3L3)

22 KN / m

36 KN / m

D
162

/ 3 ) x Base x h2
( 2 / 3 ) x 6 x 99

A3 = ( 2 / 3 ) x Base x h3
= ( 2 / 3 ) x 6 x 162
= 648
a3 = 3 m

) + 2MC ( 6 / I + 6 / I ) + MD ( 6 / I ) = - [ 6 x 396 x 3 / ( I x 6 ) ] - [ 6 x 648 x 3 / ( I x 6) ]

4MC + 6MD = - [ 1188 ] - [ 1944 ]

= - 522

As MD = 0

= - 522

.( 1 )

.( 2 )

= - 522

.( 2 )
ng Value of MB from Equation ( 2 ) into Equation ( 1 )

4MC ) + MC = - 522
16 MC + MC = - 522

104.4 KNm

ng Value of MC into Equation ( 1 )

- 104.4 ) = - 522

04.4 KNm
36 KN / m

22 KN / m

6m
104
(-)

99

C
104

6m

162 D
(+)

(-)

110
(+)
5.0

(-)

6 m Distance
104 KNm
3 m Distance
(?)
52
( 162 - 52 = 110 KNm )
Moment in span BC = ( 104 - 99 ) = 5 KNm

104
125.33

MB = - 104 KNm

66
90.67
125.33

36 x 6 x 3 = -104

VA x 6 -36 x 6 x 3 = -104

X dist.

VA = 90.67 KN

6 - X dist.

VA + VB = 36 x 6

125.33 X =90.67 ( 6 - X )

VB = 216 - 90.67

216 X = 544.02

VB = 125.33 KN

X = 2.52 m

MB = - 104 KNm
90.67 x 12 + VC x 6 - 22 x 6 x 3 - 36 x 6 x 9 = -104
Vc = 191.33 KN
VC + 125.33 = 191.33
VC = 66 KN
MC = - 104 KNm
90.67 x 12 + VB x 6 - 36 x 6 x 9 - 22 x 6 x 3 = - 104
VB = 191.33 KN
VB + 125.33 = 191.33
VB = 66 KN

90.67
125.33

2 = 110 KNm )

Design of Cantilever Beam


Span = 3 m
Characteristic U.D.L. = 12 KN /m
Assume that sufficient safety against overturning is there
( Given )
and reinforcement anchorages are also available.
Material M15 grade concrete
mild steel reinforcement.
Solution : Assume an initial trial section of 230 mm x 600 mm overall depth to consider self-weight.
The self-weight = 0.23 x 0.6 x 25 = 3.45
KN / m
Total load = 12 + 3.45 = 15.45 KN / m
Factored load = 1.5 x 15.45 = 23.2 KN / m
Factored S.F. = w x = 23.2 x 3 = 69.6 KN
Factored B.M. = w x 2 / 2 = 23.2 x 32 / 2 = 104.4 KNm
= ( 104.4 x 106 ) / ( 2.22 x 230 )
( From Table 6-3 )
= 452
mm
Using one layer of 20 mm O bars and overall depth of 550 mm
104.4 KNm
d = 550 - 25 - 10 = 515 mm
OR
Mu / b d2 == 104.4 x 106 / ( 230 x 5152)
Mu,lim = 2.22 x 230 x 5152 x 10-6
= 1.71
< 2.22 ( Table 6-3 )
= 135.424 KNm
Mu < Mu,lim
The section is singly reinforced ( under-reinforced )
Depth required = M / Q b

Pt = 50

1 - 1 - ( 4.6 / fck) x ( Mu / bd2 )


fy / fck

Pt = 50 1 - 1 - ( 4.6 / 15 ) x ( 1.71 )
250 / 15
= 0.93
Ast = ( 0.93 / 100 ) x 230 x 515 = 1102 mm2
Provide 4 - 20 mm O giving Ast = 4 x 314 = 1256 mm2 .
Check for development length :
Ld = 1102 / 1256 = 55 O
( From Table 7-6 )
IS 456-2000 clause 26.2.1 ( From Table 7-5 )
Development length of bars Ld = O s / 4 x bd
= O ( 0.87 x 250 x 1102 ) / ( 1256 x 4 x 1.0 )
= 47.71
O
= 47.71 x 20
= 954
mm
The bar shall extend into the support for a straight length of 954 mm . Provide
anchorage of 1200 mm . If in some case the bars are to be bent e.g. anchored in
column , the bearing stress around the bend has to be checked as discussed
in the next example .
Check for shear :

Vu = 69.6 KN
Actual shear strength v = Vu / bd
= 69.6 x 103 / ( 230 x 515 )
= 0.588
N / mm2
100 x As / b d = ( 100 x 1256 ) / ( 230 x 515 )
( From IS 456-2000 , table 19 table 7-1 )
= 1.06
0.25 difference
design ( permissible ) shear strength c = 0.61
N / mm2 > v
0.19 difference
Provide minimum shear reinforcement . For 230 mm wide beam ,
From IS 456-2000 clause 26.5.1.6
Spacing of minimum shear reinforcement using 6 O stirrups
= 0.87 Asv fy / 0.4 b
= 0.87 x 56 x 250 / 0.4 x 230
= 132.4
mm
spacing should not exceed
( i ) 450 mm
( ii ) 0.75 d = 0.75 x 515 = 386 mm
( iii ) 132.4 mm ( minimum )
( iv ) 569.2 mm ( designed )
Provide 6 mm O two-legged stirrups @ 130 mm c/c
Check for deflection :
Basic span / d ratio = 7
100 Ast / b d = 100 x 1256 / 230 x 515 = 1.06
modification factor = 1.4
IS 456-2000 clause 23.2.1 fig-4 ,for tension reinforcement
Span / d permissible = 7 x 1.4 = 9.8
Actual span / d = 3000 / 515 = 5.83 < 9.8
( safe )
Check for cracking (spacing of bars ) :
Clear distance between bars
= ( 230 - 50 - 4 x 20 ) / 3 = 10 mm
Maximum clear distance permitted
= 300 mm ( cracking - table 8-1 , IS 456-2000 , table 15 ) ( safe )
The design beam is shown in fig.
4 -20 O
v v v v
550
Fixed end
support
1200
6 O @ 130 c/c

2-12 O
3000
Stirrups 6 O @ 130 c/c

v
230

Design of Cantilever Beam


A cantilever rectangular bracket projects from a column of size 230 mm x 500 mm
in the direction of 500 mm for a length of 3 m
Factored load of 20 KN / m inclusive of self - weight
Material M15 grade concrete
HYSD reinforcement of grade Fe415 .
Solution : Vu = w x = 20 x 3 = 60 KN
Mu = w x 2 / 2 = 20 x 32 / 2 = 90 KNm
( a ) Moment steel :
Take size of Beam 230 mm x 550 mm overall.
Assuming 16 mm diameter bars in one layer
d = 550 - 25 ( Cover ) -8
OR
= 517
mm
2
Mu / b d =90 x 106 / ( 230 x 5172)

Mu,lim =

2.07 x 230 x 5172 x 10-6

= 127.256 KNm

Mu < Mu,lim
= 1.46
< 2.07 ( Table 6-3 )
The section is singly reinforced ( under-reinforced ) Beam
Pt = 50

1 - 1 - ( 4.6 / fck) x ( Mu / bd2 )


fy / fck

Pt = 50 1 - 1 - ( 4.6 / 15 ) x ( 1.46 )
415 / 15
= 0.464
Ast = ( 0.464 / 100 ) x 230 x 517 = 552 mm 2
Provide 16 mm O - 3 No. = 603 mm2.
Use 2- 10 mm O as bottom anchor bars .
( b ) Check for development length :
IS 456-2000 clause 26.2.1
Development length of bars Ld = O s / 4 x bd
Stress in bar s =

0.87 x fy = 0.87 x 415 x 552 / 603 =

330.5

N / mm2

bd = 1.6 x 1 = 1.6 N / mm2


IS 456-2000 clause 26.2.1 ( From Table 7-5 )
Development length of bars Ld = O s / 4 x bd
= O 330.5 / ( 4 x 1.6 )
= 51.64
O
= 51.64 x 16
= 826
mm
The arrangement of anchoring the bars is shown in fig. and is equivalent to an anchorage
of 1127 mm. The bearing stress inside the bend is now checked .
O = 16 mm
a = 82 mm for internal bar ( centre to centre distance between bar )
( = ( 230 - 50 - 3 x 16 ) / 2
= 25 + 16 = 41 mm for external bar .
( = cover + dia of bar )
(

33

The bearing stress is critical in external bar . Check for this stress for a = 41 mm.
Design bearing stress = 1.5 x fck / [1 + ( 2O/a ) ]
= 1.5 x 15 / [ 1 + ( 2 x 16 / 41 ) ]
550
= 22.5 / 1.78

33

= 12.6404 N / mm2
At the centre of bend , the anchorage available = 279 + 147 = 426 mm
Stress in bar at centre of the bend = 330.5 x ( 826 - 426 ) / 826
= 160
N / mm2
Fbt = 160 x 201 x 10-3 = 32.16 KN
( area of 16 O bar = 201 mm2 )
Bearing stress = Fbt / r O
= 32.16 x 103 / 180 x 16
= 11.16 N / mm2 < 12.64 N /mm2
The arrangement is thus satisfactory.
( c )Check for shea
Vu = 60 KN

..( safe )

Actual shear strength v = Vu / bd


= 60 x 103 / ( 230 x 517 )
= 0.505
N / mm2
100 x As / b d = ( 100 x 603 ) / ( 230 x 517 )
( From IS 456-2000 , table 19 table 7-1 )
= 0.507
0.25 difference
design ( permissible ) shear strength c = 0.46
N / mm2 < v
0.243 difference
shear design is necessary .
Vus = Vu - c b d
Vus = ( 0.505 - 0.46 ) x 230 x 517 x10 -3
At support ,
OR
= 60 - 0.46 x 230 x 517 x 10-3
= 5.35
= 60 54.6986
= 5.3
KN
Using 6 mm O ( mild steel ) Two legged stirrups , Asv = 28 x 2 = 56 mm2 .
fy = 250 N / mm2
Sv = 0.87 fy Asv d / Vus
= 0.87 x 250 x 56 x 517 / 5.3 x 103
= 1188
mm
Provide minimum shear reinforcement . For 230 mm wide beam ,
From IS 456-2000 clause 26.5.1.6
Spacing of minimum shear reinforcement using 6 O stirrups
= 0.87 Asv fy / 0.4 b
= 0.87 x 56 x 250 / 0.4 x 230
= 132.4
mm
spacing should not exceed
( i ) 450 mm
( ii ) 0.75 d = 0.75 x 517 = 388 mm

( iii ) 132.4 mm ( minimum )


( iv ) 1188 mm ( designed )
Provide 6 mm O two-legged stirrups @ 130 mm c/c
( d ) Check for deflection :
Basic span / d ratio = 7
100 Ast / b d = 100 x 603 / 230 x 517 = 0.507
modification factor = 1.2
IS 456-2000 clause 23.2.1 fig-4 ,for tension reinforcement
Span / d permissible = 7 x 1.2 = 8.4
Actual span / d = 3000 / 517 = 5.8
< 8.4
( safe )
( e ) Check for cracking (spacing of bars ) :
Clear distance between bars
= ( 230 - 50 - 3 x 16 ) / 2 = 66 mm
Minimum clear distance permitted
= hagg + 5 mm =
20 + 5 = 25 mm or 16 mm ( O of bar ) i.e. 25 mm .
Maximum clear distance permitted
= 180 mm ( cracking - table 8-1 , IS 456-2000 , table 15 ) ( safe )
The design beam is shown in fig.

295

279
r

108
r

50

3-16 O

internal radius
r = 180
550

295
150
500
Stirrups

2-10 O
3000 mm
6 mm O @ 130 mm c/c

er self-weight.
23.2 KN /m

3m

69.6 KN

104.4 KNm

230 x 5152 x 10-6

IS 456-2000 , Table 19
Table 7-1
Design shear strength of concrete , C, N / mm2
Pt = 100
Concrete grade
x As

19 table 7-1 )
0.04
?

-0.0304

x As
bxd

M15

M20

M25

M30

M35

0.15
0.25
0.50

0.28
0.35
0.46

0.28
0.36
0.48

0.29
0.36
0.49

0.29
0.37
0.50

0.29
0.37
0.50

0.75
1.00

0.54
0.60

0.56
0.62

0.57
0.64

0.59
0.66

0.59
0.67

1.25
1.50
1.75
2.00

0.64
0.68
0.71
0.71

0.67
0.72
0.75
0.79

0.70
0.74
0.78
0.82

0.71
0.76
0.80
0.84

0.73
0.78
0.82
0.86

2.25
0.71
0.81
0.85
0.88
0.90
2.50
0.71
0.82
0.88
0.91
0.93
2.75
0.71
0.82
0.90
0.94
0.96
3.00
0.71
0.82
0.92
0.96
0.99
The above given table is based on the following formula
Design shear strength c = 0.85 0.8 x fck ( 1 + 5 x - 1 )
6x
= 0.8 x fck / 6.89 Pt , but not less than 1.0

nforcement

( safe )

4-20 O
v

2-12 O

( given )

230 x 5172 x 10-6

an anchorage

( = ( 230 - 50 - 3 x 16 ) / 2 + 16 = 82 mm )
33 82 82 33

33 82 82 33

IS 456-2000 Clause 26.2.2.5 Bearing stresses at bends

16 25

The bearing stress in concrete for bends and hooks described in IS : 2502
need not be checked . The bearing stress inside a bend in any other bend
calculated as given below :
Bearing stress = Fbt / r O
Where ,
Fbt = tensile force due to design loads in a bar or group of bars,

230

r = internal radius of the bend , and


O = size of the bar or , in bundle , the size of bar of equivalent area
For Limit state method of design , this stress shall not exceed 1.5 f
Where,
fck is the characteristic strength of concrete and

a is the centre to centre distance between bars or group of bars , for a ba


of bars adjacent to the face of the member a shall be taken as the cover p
bar ( O )

19 table 7-1 )
0.08
?

-0.0778

0.505 - 0.46 ) x 230 x 517 x10 -3

KN

nforcement

( safe )

3-16 O

2-10 O
230

, N / mm2
C

M40
0.30
0.38
0.51
0.60
0.68
0.74
0.79
0.84
0.88
0.92
0.95
0.98
1.01
1+5x-1)

ss than 1.0

s at bends

ooks described in IS : 2502-1963


de a bend in any other bend shall be

or group of bars,

ar of equivalent area
hall not exceed 1.5 fck / [ ( 1+ 2O/a ) ]

or group of bars , for a bar or group


hall be taken as the cover plus size of

Design of Simply supported Doubly reinforced Beam


Span = 5 m
( simply supported rectangular beam )
super-imposed load = 40 KN / m
Beam section = 230 mm x 600 mm
( Given )
Material M15 grade concrete
HYSD reinforcement of grade Fe415
Solution : DL of beam 0.23 x 0.60 x 25 = 3.45 KN / m
super-imposed load = 40 KN / m
Total 43.45 KN / m
Factored load = 1.5 x 43.45 = 65.18 KN / m
Mu = w x 2 / 8
= 65.18 x 52 / 8
= 203.688 KNm
Vu = w x / 2
= 65.18 x 5 / 2
= 162.95 KN
( a ) Moment steel :
Assuming 20 mm diameter bars in two layer
d = 600 - 25 ( Cover ) - 20 - 10
= 545
mm
2
Mu / b d = 203.69 x 106 / ( 230 x 5452)

OR
Mu,lim =

2.07 x 230 x 5452 x 10-6

= 141.414 KNm

Mu > Mu,lim
= 2.98
> 2.07 ( Table 6-3 )
The section is Doubly reinforced ( over-reinforced )
Mu,lim = 2.07 x 230 x 5452 x 10-6
=
141.414 KNm
Mu2 = Mu - Mu,lim
=
203.69 - 141.41
=
62.28
KNm
Let the compression reinforcement be provided at an effective cover of 40 mm .
d' / d = 40 / 545
= 0.07
consider d' / d = 0.1 .
Stress in compression steel , fsc = 353 N / mm2 ( refer to table 6-6 )
Asc = Mu2 / ( fsc x ( d - d' ) )
= 62.28 x 106 / 353 ( 545 - 40 )
= 349
mm2
Corresponding tension steel
Ast2 = Asc fsc / 0.87 fy
= 349 x 353 / ( 0.87 x 415 )
= 341.2
mm2
Ast,lim = Mu,lim / ( 0.87 fy ( d - 0.42 Xu,max ) )
= 141.41 x 106 / ( 0.87 x 415 ( 545 - 0.42 x 0.48 x 545 ) )
= 141.46 x 106 / 361.05 ( 435.13 )

= 900
Ast = Ast,lim + Ast2 .

mm2

= 900 + 341.2 mm2


= 1241.2 mm2
Provide
Asc = 2-16 O = 402
mm2
Ast = 4-20 O = 1256
mm2
( all straight )
( b ) Check for development length :
As all the bars are taken into support , Mu1 may be taken as Mu .
Assume L0 = 12 O

Ld = 56 O ( From Table 7-6 )

1.3 x M1 / V + L0 Ld
1.3 x 203.69 x 106 / 162.95 x 103 +12 O 56 O
1625.02

44 O
O

36.93
mm
Oprovided = 20 mm ..( safe )
( c ) Check for shear
Vu = 162.95 KN
Actual shear strength v = Vu / bd
= 162.95 x 103 / ( 230 x 545 )
= 1.3
N / mm2
100 x As / b d = ( 100 x 1256 ) / ( 230 x 545 )
( From IS 456-2000 , table 19 table 7-1 )
= 1.0
design ( permissible ) shear strength c = 0.6
N / mm2 < v
As the ends are confined with compressive reaction , shear at distance d will be used for
checking shear at support . At 545 mm , shear is equal to
Vu = 162.95 - 0.545 x 43.45 = 139.27 KN
Vus = Vu - c b d
= 139.27 - 0.6 x 230 x 545 x 10-3 = 64.06 KN
Assuming 8 mm O two - legged stirrups , Asv = 100 mm2 , fy = 415 N / mm2 .
Sv = 0.87 fy Asv d / Vus
= 0.87 x 415 x 100 x 545 / 64.06 x 103
= 307
mm
From IS 456-2000 clause 26.5.1.6
Spacing of minimum shear reinforcement using 8 mm O stirrups
= 0.87 Asv fy / 0.4 b
= 0.87 x 100 x 415 / 0.4 x 230
= 392.4
mm
spacing should not exceed
( i ) 450 mm
( ii ) 0.75 d = 0.75 x 545 = 408 mm

( iii ) 392.4 mm ( minimum )


( iv ) 307 mm ( designed )
Minimum shear reinforcement of 8 mm O @ 390 mm c/c will be used .
At support provide 8 mm O @ 300 mm c/c .
Shear resistance of beam with minimum shear reinforcement
= ( 0.87 fy Asv d / Sv )+ c b d
= (0.87 x 415 x 100 x 545 x 10-3 / 390 ) + 0.6 x 230 x 545 x 10-3
= 50.45
+
75.21
= 125.66 KN
Designed shear reinforcement is required from face of the support upto the
distance equal to
162.95 - 125.66 / 43.45 = 0.858 m
No. of stirrups = ( 858 / 300 ) + 1 = 3.86 say 4
Provide 8 mm O two-legged stirrups @ 300 mm c/c upto 4 no. from face of the support
and 8 mm O @ 390 mm c/c in remaining central portion .
( d ) Check for deflection :
Basic span / d ratio = 20
100 Ast / b d = 100 x 1256 / 230 x 545 = 1.0
modification factor = 0.97
IS 456-2000 clause 23.2.1 fig-4 ,for tension reinforcement
100 Asc / b d = 100 x 402 / 230 x 545 = 0.32
modification factor = 1.08
IS 456-2000 clause 23.2.1 fig-5 ,for compression reinforcement
Span / d permissible = 20 x 0.97 x 1.08 = 20.95
Actual span / d = 5000 / 545 = 9.17
( safe )
( d ) Check for cracking (spacing of bars ) :
Clear distance between bars
= ( 230 - 50 - 4 x 20 ) / 3 = 10 mm
Minimum clear distance permitted
= 20 + 5 = 25 mm or 20 mm ( O of bar ) i.e. 25 mm .
= hagg + 5 mm
Maximum clear distance permitted
= 180 mm ( cracking - table 8-1 , IS 456-2000 , table 15 ) ( safe )
The design beam is shown in fig.
2-16 O

2-16 O

600

545
4- 20 O ( straight )

600

230

5000 c/c

DIA
NO.
SPA.

8O
4
300

8O
rest
390

600
bearing

8O
4
300

4- 20 O

Table 6-6
STRESS IN COMPRESSION REINFORCEMENT fsc ,
N / mm2 IN DOUBLY REINFORCED BEAMS
d'/d
fy N / mm2
0.05
0.1
0.15
0.2
250
217
217
217
217
415
355
353
342
329
500
424
412
395
370
230 x 5452 x 10-6

550

458

441

419

380

If Mu > Mu,lim : design the section either increasing the dimensions of se


Over reinforced section ( doubly-reinforced beam ).The additional momen
needed is obtained by providing compression( top ) reinforcement and ad
reinforcement. Mu2 = Mu - Mu,lim as explained below .
Ast,lim = Mu,lim / ( 0.87 fy ( d - 0.42 Xu,max ) )
Asc = Mu2 / ( fsc x ( d - d' ) )
Ast2 = Asc fsc / 0.87 fy
Ast = Ast,lim + Ast2 .
If Xu < Xu,max the section is under-reinforced ( singly reinforced )
If Xu = Xu,max the section is balanced
If Xu > Xu,max the section is over-reinforced ( doubly reinforced )
where ,
Xu,max = 0.53 x d ( for Fe250 mild steel )
Xu,max = 0.48 x d ( for Fe415 HYSD steel )
Xu,max = 0.46 x d ( for Fe500 HYSD steel )
Xu,max = 0.44 x d ( for Fe550 HYSD steel )
Xu =( 0.87 fy Ast ) / ( 0.36 fck b )

19 table 7-1 )

l be used for

he support

nforcement

on reinforcement

( safe )

easing the dimensions of section or deign as


am ).The additional moment of resistance M u2
top ) reinforcement and additional tensile
low .

ngly reinforced )

ubly reinforced )

Design of Simply supported Singly reinforced Beam


Span = 6 m
( simply supported rectangular beam )
characteristic load = 20 KN / m inclusive of its self-weight
Beam section = 230 mm x 600 mm
Material M15 grade concrete
( Given )
HYSD reinforcement of grade Fe415
The beam is resting on R.C.C. columns.
Solution : Factored load = 1.5 x 20 = 30 KN /m.
Mu = w x 2 / 8
= 30 x 62 / 8
= 135
Vu = w x / 2

KNm

= 30 x 6 / 2
= 90

KN

( a ) Moment steel :
Assuming 20 mm diameter bars in one layer
d = 600 - 25 ( Cover ) -10
= 565
mm

OR

Mu / b d2 = 135 x 106 / ( 230 x 5652)


= 1.84

< 2.07 ( Table 6-3 )

Mu,lim =

2.07 x 230 x 5652 x 10-6

= 151.983 KNm
Mu < Mu,lim

The section is singly reinforced ( under-reinforced )


Pt = 50
Pt = 50

1 - 1 - ( 4.6 / fck) x ( Mu / bd2 )


fy / fck
1 - 1 - ( 4.6 / 15 ) x ( 1.84 )
415 / 15

= 0.614
Ast = ( 0.614 / 100 ) x 230 x 565 = 798 mm 2
As per IS 456-2000 clause 26.5.1.1 ( a )
Minimum steel , As = ( 0.205 / 100 ) x 230 x 565 = 266 mm 2
From Table 6-4
Ast,lim = 0.72 / 100 x 230 x 565 = 936 mm2.
Provide 16 mm O - 4 No. = 804 mm2.
Let 2 bars are bent at 1.25 D = 1.25 x 600 = 750 mm , from the face of the support .
( b ) Check for development length :
( 1 ) A bar can be bent up at a distance greater than L d = 56 O ( Table 7-6 )

From the centre of the support , i.e. 56 x 16 = 896 mm .


in this case , the distance is ( 3000 - 750 ) = 2250 mm .( safe )
( 2 ) For the remaining bars , Ast = 402 mm2
Mu1 = 0.87 fy Ast d ( 1 - [( fy Ast ) / ( fck b d ) ] )
=
=
=
Vu =

0.87 x 415 x 402 x 565 ( 1 - [ ( 415 x 402 ) / ( 15 x 230 x 565 ) ] ) x 10 -6


82.01
x
0.91
74.99
KNm
L0 = 12 O ( assume )
90 KN ,

As the reinforcement is confined by compressive reaction


1.3 x M1 / V + L0 Ld
1.3 x 74.99 x 106 / 90 x 103 +12 O 56 O
1083.19

44 O
O

24.62
mm
Oprovided = 16 mm ..( safe )
( c ) Check for shear :
At support , Vu = 90 KN
As the ends of the reinforcement are confined with compressive reaction , shear at distance d
will be used for checking shear at support.
Vu = 90 - 0.565 x w = 90 - 0.565 x 30 = 73.05 KN
Actual shear strength v = Vu / bd
= 73.05 x 103 / ( 230 x 565 )
= 0.562
N / mm2
100 x As / b d = ( 100 x 402 ) / ( 230 x 565 )
( From IS 456-2000 , table 19 table 7-1 )
= 0.309
0.25 difference
design ( permissible ) shear strength = 0.376
N / mm2 < v
0.191 difference
c

shear design is necessary .


Vus = Vu - c b d
At support ,

OR

= 73.05 - 0.376 x 230 x 565 x 10-3


= 73.05 48.86
= 24.19
KN
Capacity of bent bars to resist shear
( 0.87 fy Asv sin )
= 2 x 201 x 0.87 x 415 x sin 45 x 10-3
= 102.6
KN
Bent bars share 50 % = 12.09 KN
Stirrups share 50 % = 12.09 KN
Using 6 mm O ( mild steel ) Two legged stirrups , Asv = 28 x 2 = 56 mm2 .
Sv = 0.87 fy Asv d / Vus
= 0.87 x 250 x 56 x 565 / 12.09 x 103
= 569.2
mm

Vus = ( 0.562 - 0.376 ) x 230 x 565

= 24.17

From IS 456-2000 clause 26.5.1.6


Spacing of minimum shear reinforcement using 6 O stirrups
= 0.87 Asv fy / 0.4 b
= 0.87 x 56 x 250 / 0.4 x 230
= 132.4
mm
spacing should not exceed
( i ) 450 mm
( ii ) 0.75 d = 0.75 x 565 = 423 mm
( iii ) 132.4 mm ( minimum )
( iv ) 569.2 mm ( designed )
Provide 6 mm O two-legged stirrups @ 130 mm c/c
At 1.25 D = 750 mm from face of the support where contribution of bent bars is not available
Vu = 90 - 0.75 x w = 90 - 0.75 x 30 = 67.5 KN
Vus = Vu - c b d
= 67.5 - 0.376 x 230 x 565 x 10-3
= 67.5 48.86
= 18.64
KN
Provide minimum 6 mm O M.S. two -legged stirrups @ 130 mm c/c throught the beam.
( d ) Check for deflection :
Basic span / d ratio = 20
100 Ast / b d = 100 x 804 / 230 x 565 = 0.62
modification factor = 1.1

IS 456-2000 clause 23.2.1 fig-4 ,for tension reinforcement

Span / d permissible = 20 x 1.1 = 22


Actual span / d = 6000 / 565 = 10.62

( safe )

( d ) Check for cracking (spacing of bars ) :


Clear distance between bars
= 230 - 50 - 2 x 16 = 148 mm
Minimum clear distance permitted
= hagg + 5 mm =
20 + 5 = 25 mm or 16 mm ( O of bar ) i.e. 25 mm .
Maximum clear distance permitted
= 180 mm

( cracking - table 8-1 , IS 456-200 , table 15 )

( safe )

The design beam is shown in fig.


2-10 O

2-10 O

600

565
4- 16 O ( 2 straight +
2 bent )

750

230
6000 c/c

4- 16 O

6 mm O @ 130 mm c/c

Consider width of the beam equal to 230 mm. The depth may be assumed
as 1 / 10 to 1 / 8 of the span.
To find steel area
( 1 ) For a given ultimate moment ( also known as factored moment )
and assumed width of section , find out d from equation
d = Mu / Qlim b
This is a balanced section and steel area may be found out from table P

( 2 ) For a given factored moment ,width and depth of section .


Obtain Mu,lim = Qlim bd2 .
If Mu < Mu,lim :

design as under-reinforced section (singly reinforced beam) as exp

Pt = 50
If Mu = Mu,lim :

230 x 5652 x 10-6

1 - 1 - ( 4.6 / fck) x ( Mu / bd2 )


fy / fck

design as balanced section as explained in ( 1 ).

If Mu > Mu,lim : design the section either increasing the dimensions of section or d
Over reinforced section ( doubly-reinforced beam ).The additional moment of resist
needed is obtained by providing compression( top ) reinforcement and additional te
reinforcement. Mu2 = Mu - Mu,lim as explained below .
Ast,lim = Mu,lim / ( 0.87 fy ( d - 0.42 Xu,max ) )
Asc = Mu2 / ( fsc x ( d - d' ) )
Ast2 = Asc fsc / 0.87 fy
Ast = Ast,lim + Ast2 .
If Xu < Xu,max the section is under-reinforced ( singly reinforced )
If Xu = Xu,max the section is balanced
If Xu > Xu,max the section is over-reinforced ( doubly reinforced )
where ,
Xu,max = 0.53 x d ( for Fe250 mild steel )
Xu,max = 0.48 x d ( for Fe415 HYSD steel )
Xu,max = 0.46 x d ( for Fe500 HYSD steel )
Xu,max = 0.44 x d ( for Fe550 HYSD steel )
Xu =( 0.87 fy Ast ) / ( 0.36 fck b )
Table 6-2
Limiting Moment of Resistance and Reinforcement

Index for Singly Reinforced Rectangular Sections

fy , N / mm2

250

415

500

550

Mu,lim / fck b d2

0.148

0.138

0.133

0.129

Pt,lim fy / fck

21.93

19.86

19.03

18.2

Table 6-3
Limiting Moment of resistance factor Q lim, N / mm2
For singly reinforced rectangular sections

fck
N
/ mm2

fy, N / mm2

15
20
25

250
2.22
2.96
3.70

415
2.07
2.76
3.45

500
2.00
2.66
3.33

550
1.94
2.58
3.23

30

4.44

4.14

3.99

3.87

Table 6-4
Limiting Percentage of Reinforcement Pt,lim

at distance d

For singly reinforced rectangular sections


fck
N
2
/ mm

19 table 7-1 )
0.11
?

fy, N / mm2

15
20

250
1.32
1.75

415
0.72
0.96

500
0.57
0.76

550
0.50
0.66

25
30

2.19
2.63

1.20
1.44

0.95
1.14

0.83
0.99

-0.084
IS 456-2000 Clause 26.5 Requirements of Reinforcement for

0.562 - 0.376 ) x 230 x 565 x10

KN

-3

Structural Members
26.5.1 Beams
26.5.1.1 Tension Reinforcement
a ) Minimum reinforcement - The minimum area of tension reinforcement
shall not be less than that given by the following :
As / b d = 0.85 / fy
where ,
As = minimum area of tension reinforcement ,
b = breadth of the beam or the breadth of the web of T- beam ,
d = effective depth , and
fy = characteristic strength of reinforcement in N / mm 2 .
b ) Maximum reinforcement - The maximum area of tension
reinforcement shall not exceed 0.04 b D.

not available

Minimum steel %
For mild steel
100 As / b d = 100 x 0.85 / 250 = 0.34
For HYSD steel , Fe415 grade
100 As / b d = 100 x 0.85 / 415 = 0.205
For HYSD steel , Fe500 grade
100 As / b d = 100 x 0.85 / 500 = 0.17

For checking development length , l0


may be assumed as 8 O for HYSD
bars ( usually end anchorage is not
provided ) and 12 O for mild steel ( U
hook is provided usually whose
anchorage length is 16 O.

check for development length


IS 456-2000 clause 26.2.1
Development length of bars Ld = O s / 4 x bd
IS 456-200 clause 26.2.1.1
Table 7-5
Design bond stress (bd ) for plain bars in tension

orcement

( safe )

Concrete
grade
(

M15

M20

M25

M30

M35

M40

bd N / mm 1.0

1.2
1.4
1.5
1.7
1.9
Note-1 : bd shall be increased by 25 % for bars in compression
Note-2 : In case of deformed bars confirming to IS : 1786-1985 ,
the value of bd shall be increased by 60 %.
For mild steel Fe250

s = 0.87 x fy

s = 0.67 x fy
For Fe415
IS 456-2000 clause 26.2.3.3
Ld M1 / V + L0
L0 = effective depth of the members or 12 O , whichever is greater
if ends of the reinforcement are confined by a compressive
reaction M1 / V increased by 30 % ( 1.3 M1 / V )
Table 7-6
Development length for single mild steel bars

fy N /
mm2
250
415

Tension bars
M15
M20
55 O
26 O
56 O
47 O

Compression bars

M15
44 O
45 O

M20
37 O
38 O

500

69 O

58 O

54 O

46 O

check for shear


IS 456-2000 , Table 19
Table 7-1
Design shear strength of concrete , C, N / mm2
Concrete grade
M15
M20
M25
M30
M35
M40
0.15
0.28
0.28
0.29
0.29
0.29
0.30
0.25
0.35
0.36
0.36
0.37
0.37
0.38
0.50
0.46
0.48
0.49
0.50
0.50
0.51
0.75
0.54
0.56
0.57
0.59
0.59
0.60
1.00
0.60
0.62
0.64
0.66
0.67
0.68
1.25
0.64
0.67
0.70
0.71
0.73
0.74
1.50
0.68
0.72
0.74
0.76
0.78
0.79
1.75
0.71
0.75
0.78
0.80
0.82
0.84
2.00
0.71
0.79
0.82
0.84
0.86
0.88
2.25
0.71
0.81
0.85
0.88
0.90
0.92
2.50
0.71
0.82
0.88
0.91
0.93
0.95
2.75
0.71
0.82
0.90
0.94
0.96
0.98
3.00
0.71
0.82
0.92
0.96
0.99
1.01
The above given table is based on the following formula
Design shear strength c = 0.85 0.8 x fck ( 1 + 5 x - 1 )
6x
= 0.8 x fck / 6.89 Pt , but not less than 1.0

Pt = 100 x As
bxd

IS 456-2000 , Table 20
Table 7-2
Maximum shear stress , C, N / mm2
Concrete
grade

( c )max N/mm

M15
2.5

M20
2.8

M25
3.1

M30
3.5

M35
3.7

M40
4.0

IS 456-2000 Clause 26.5.1.6 Minimum shear reinforcement


Minimum shear reinforcement in the form of stirrups shall be provided
Asv / b sv 0.4 / 0.87 fy
such that :
where,
Asv = total cross-sectional area of stirrup legs effective in shear ,
Sv = stirrup spacing along the length of the member ,
b = breadth of the beam or breadth of the web of flanged beam , and
fy = characteristic strength of the stirrup reinforcement in N / mm 2
which shall not be taken greater than 415 N / mm 2 .

check for deflection


Basic values of span to effective depth ratios for spans upto 10 m :
cantilever
7
simply supported
20
continuous
26
For spans above 10 m, the values in (a) may be multiplied by
10 / span in metres , except for cantilever in which case deflection
calculations should be made.
check for cracking
IS 456-2000 26.3.3 Maximum distance between bars in tension

Table 15 Clear distance Between Bars ( Clause


26.3.3 )
% redistribution to or from section considered

fy

-30

N / mm2
250
415
500

mm
215
125
105

-15
0
+15
+30
Clear distance between bars
mm
260
155
130

mm
300
180
150

mm
300
210
175

mm
300
235
195

may be assumed

ed moment )

out from table P t,lim , SP : 16 ,2.3

gly reinforced beam) as explained below.

d in ( 1 ).

dimensions of section or deign as


additional moment of resistance M u2
orcement and additional tensile

sion reinforcement

nsion

is greater

ll be provided

ve in shear ,

anged beam , and

ent in N / mm 2

Design of slender ( Long ) columns ( with biaxial bending )


Size of column 400 x 300 mm
Column is restrained against sway.
Concrete grade M 30
Characteristic strength of reinforcement 415 N/mm 2
Effective length for bending parallel to larger dimension ex = 6.0 m
Effective length for bending parallel to shorter dimension ey = 5.0 m
Unsupported length = 7.0 m
Factored load 1500kN
Factored.moment in the direction of larger dimension = 40 kNm at top and 22.5 KNm at bottom.
Factored.moment in the direction of shorter dimension = 30 kNm at top and 20 KNm at bottom.
Solution : About x axis :
1 and 2 are the same
For Beam :
bf = l 0 / 6 + b w + 6 D f
= 0.7 x 5000 / 6 + 230 + 6 x 120
= 1533.33333
bf / bw = 1533.3 / 230 = 6.67
Df / D = 120 / 600 = 0.2
Kt from chart 88 , SP : 16 = 2.07
Beam stiffness
Kb = 1.5 x Ib / l
= 1.5 x ( 2.07 x ( 1 / 12 ) x 230 x 6203 ) / 5000
=
2836699 mm3
Column stiffness
Kc = Ic / l = 1/12 x 230 x 4003 / 7000
= 175238.095 mm3
1 = 2 = Kc / ( Kc + Kb )
= 2 x 175238 / 2 ( 175238 + 2836699 )
=
0.0582
as per IS 456-2000 fig. 27
lef / l = 1.035 < 1.2
.consider 1.2
lex = 1.2 x unsupported length
= 1.2 ( 7000 - 620 )
= 7656
mm
lex / D = 7656/400
= 19.14
> 12
The column is long about x direction.
About Y axis :

Beam stiffness
Kb = 1.5 x Ib / l
= 1.5 x ( ( 1 / 12 ) x 230 x 4203 ) / 5000
= 426006
mm3
Column stiffness
Kc = Ic / l = 1/12 x 400 x 2303 / 7000
= 57938.0952 mm3
1 = 2 = Kc / ( Kc + Kb )
= 2 x 57938.1 / 2 ( 57938.1 + 426006 )
= 0.1197
as per IS 456-2000 fig. 27
lef / l = 1.06 < 1.2
.consider 1.2
ley = 1.2 x unsupported length
= 1.2 ( 7000 -420 )
= 7896
mm
ley / b = 7896/300
= 26.32
> 12
The column is long about Y direction.
The column is bent in double curvature. Reinforcement will be distributed equally on four sides.
ex / D = 6000 / 400
= 15.0
> 12
ey / b = 5000 / 300
= 16.7
> 12
Therefore the column is slender about both the axes.
Additional moments
Max = ( Pu D / 2000 ) x ( ex / D )2
= ( 1500 x 400 / 2000 ) x (15)2 x 10-3
= 67.5
KNm
May = ( Pu b / 2000 ) x ( ey / b )2
= ( 1500 x 300 / 2000 ) x (16.7)2 x 10-3
= 62.75
KNm
The above moments will have to be reduced in accordance with clause 39.7.1.1 of the IS 456-2000
but multiplication factors can be evaluated only if the reinforcement is known.
For first trial , assume p = 3.0 ( with reinforcement equally on all the four sides ).
Ag = 400 x 300
Puz = 0.45 fck Ac + 0.75 f
= 120000
mm2
From chart 63 , puz / Ag = 22.5 N/mm2
Puz = 22.5 x 120000 x 10-3
= 2700
KN
Calculation of Pb :

Ac = 400 x 300 - 120000*3/100


OR

= 116400 mm2
Puz = 0.45 x 30 x 116400 x 10-3 + 0.75 x 4
= 1571.4
+
= 2692

KN

Assuming 25 mm dia bars with 40 mm cover


d' / D ( about xx-axis ) = 52.5 / 400 = 0.13 use d' / D = 0.15
d' / D ( about yy-axis ) = 52.5 / 300 = 0.18 use d' / D = 0.2
From Table 60 , SP 16
Pb ( about xx-axis ) = ( k1 + k2 p / fck ) fck b D
Pbx = ( 0.196 + 0.203 x 3 /30 ) 30 x 300 x 400 x10-3
= 779
KN
Pb ( about yy-axis ) = ( k1 + k2 p / fck ) fck b D
Pby = ( 0.184 + 0.028 x 3 /30 ) 30 x 300 x 400 x10-3
= 672
KN
Kx = ( Puz - Pu ) / ( Puz - Pbx )
= ( 2700 - 1500 ) / ( 2700 - 779 )
= 1200 / 1921
= 0.625
Ky = ( Puz - Pu ) / ( Puz - Pby )
= ( 2700 - 1500 ) / ( 2700 - 672 )
= 1200 / 2028
= 0.592
The additional moments calculated earlier , will now be multiplied by the above values of k .
Max = 67.5 x 0.625 = 42.2 KNm
May = 62.75 x 0.592 = 37.15 KNm
The additional moments due to slenderness effects should be added to the initial moments
after modifying the initial moments as follows ( see note 1 under 39.7.1 of the code IS 456-2000 )
Mux = ( 0.6 x 40 - 0.4 x 22.5 ) =
15.0
KNm
Muy = ( 0.6 x 30 - 0.4 x 20 ) =
10.0
KNm
The above actual moments should be compared with those calculated from minimum eccentricity
consideration ( see 25.4 of the code ) and greater value is to be taken as the initial moment for
adding the additional moments.
ex = ( / 500 ) + ( D / 30 ) =

( 7000 / 500 ) + ( 400 / 30) = 27.3333

ey = ( / 500 ) + ( b / 30 ) =
( 7000 / 500 ) + ( 300 / 30) = 24
Moments due to minimum eccentricity :
Mux = 1500 x 27.33 x 10-3 = 41.0 KNm > 15.0 KNm
Muy = 1500 x 24 x 10-3 = 36.0 KNm > 10.0 KNm
Total moments for which the column is to be designed are :
Mux = 41.0 + 42.2 = 83.2
KNm
Muy = 36.0 + 37.15 = 73.15
KNm
The section is to be checked for biaxial bending
Pu / fck b D = 1500 x 103 / ( 30 x 300 x 400 )

= 0.417
p / fck = 3 / 30 = 0.10
referring to chart 45 (d' / D = 0.15 ) ,
Mu / fck b D2 = 0.104
Mux1 = 0.104 x 30 x 300 x 4002
= 149.8

KNm

referring to chart 46 (d' / D = 0.2 ) ,


Mu / fck b D2 = 0.096
Muy1 = 0.096 x 30 x 400 x 3002
= 103.7
KNm
Mux / Mux1 = 83.2 / 149.8 =

0.56

Muy / Muy1 = 73.15 / 103.7 = 0.71


Pu / Puz = 1500 / 2700 = 0.56
referring to chart 64 , the maximum allowable value of Mux / Mux1 corresponding to the
above values of Muy / Muy1 and Pu / Puz is 0.58 which is slightly higher than the actual
value of 0.56 . The assumed reinforcement of 3.0 % is therefore satisfactory.
for Pu / Puz = 0.56 ,
n = 1.602
From , IS 456-2000 , Clause 39.6
n = 1.602
check :
Mux

Mux1
1.602
( 0.56 )
0.395
= 0.972

+
+

Muy
Muy1

0.2 difference

0.34

0.04 difference

1.602
( 0.71 )
0.577
..( O.K.)

As = p x b x D / 100 = 3.0 x 300 x 400 / 100


= 3600
mm2
Provide 25 mm diameter bar ( 3600 / 491 ) = 8 no. = 3928 mm 2
Ties : O min = 25 / 4
= 6.25
Provide 8 mm O M.S. ties.
Spacing should not exceed lesser of
( i ) 300 mm
( ii ) 16 x 25 = 400 mm
300

500

( iii ) 48 x 8 = 384 mm
Provide 8 mm O M.S. ties @ 300 mm c/c

300

Note that the distance between corner bars in one face is more than 48 O tr
( 500 - 80 -25 = 395 > 48 x 8 = 384 ). Therefore two sets of closed ties shall be used .
Note that if this distance would be less than ( 48 x 8 ) mm , open ties for internal bars would be
sufficient.
Design of slender ( Long ) columns ( with Uniaxial bending )
Size of column 230 x 450 mm
Column of a braced frame
Concrete grade M 20
Characteristic strength of reinforcement 415 N/mm 2 HYSD reinforcement
Unsupported length in both the direction = 5.0 m
Factored load Pu = 1000 kN
Factored.moment in the direction of larger dimension M uxx = 80 kNm at top and 60 KNm at bottom.
Factored.moment in the direction of shorter dimension M uyy = 40 kNm at top and 30 KNm at
bottom.
The column is bent in double curvature and is slender about both the axis.
The slenderness ratios ex / Ixx and ey / Iyy are respectively 13.2 and 15.6
Assume that the moments due to minimum eccentricities about both the axes are less than
applied moments.
Solution : Assume adjustment factor k = 0.8 for the first trial .
Additional moments
Max = ( Pu D / 2000 ) x ( ex / D )2
= ( 1000 x 450 / 2000 ) x (13.2 )2 x 10-3
= 39.2
KNm
May = ( Pu b / 2000 ) x ( ey / b )2
= ( 1000 x 230 / 2000 ) x (15.6 )2 x 10-3
= 28
KNm
About XX
Pu = 1000
KN
Muxx =
Mi =
=
=
=

Mi + k x Max
0.6 Mu2 + 0.4 Mu1
0.6 x 80 - 0.4 x 60
48 - 24
24 KNm
< 0.4 x 80 = 32 KNm
Mi = 32 KNm

Take ,
Note that Mu1 is considered negative as the column bends in double curvature .
Mu,xx = 32 + 0.8 x 39.2
= 63.36
KNm.

< 80 KNm

Mu,xx = 80 KNm
Take ,
About YY
Pu = 1000
KN
Muyy = Mi + k x May
Mi =
=
=
=

0.6 Mu2 + 0.4 Mu1


0.6 x 40 - 0.4 x 30
24 - 12
12 KNm
< 0.4 x 40 = 16 KNm
Mi = 16 KNm

Take ,
Mu,yy = 16 + 0.8 x 28
= 38.4
KNm.
Mu,yy = 40 KNm
Take ,
Finally design the column for
Pu = 1000 KN

< 40 KNm

Mu,xx = 80 KNm
Mu,yy = 40 KNm
For the first trial , assume uniaxial bending about y axis for the following values .
P'u = 1000 KN
M'uy = ( 230 / 450 ) ( 80 + 40 ) =
61.33
KNm
d' / D = 50 / 230 = 0.22
Say 0.2
P'u / fck b D = 1000 x 103 / 20 x 230 x 450 = 0.48
M'uy / fck b D2 = 61.33 x 106 / 20 x 450 x 2302 = 0.129
p / fck = from chart 46 , SP : 16 =
0.162
p = 0.162 x 20
= 3.24
Asc = ( 3.24 /100 ) x 230 x 450
= 3353
mm2
Assumptions made above for k = 0.8 and for uniaxial moment to find out first trial steel are
usually conservative . Let us try 4 - 25 O + 4 - 20 O = 3220 mm 2
Now check the assumed section as follows :
Puz = 0.45 fck Ac + 0.75 fy Asc
Ac = 230 x 450 - 3220
= 100280
mm2
Puz = 0.45 x 20 x 100280 x 10-3 + 0.75 x 415 x 3220 x 10-3
= 902.52
+
1002.225
= 1904.7
KN
Pb = ( k1 + k2 p / fck ) fck b D
Asc = ( p / 100 ) x b x D
p / fck = ( 3220 x 100 ) / ( 230 x 450 x 20 )

= 0.156
For d' / D = 0.2 , k1 = 0.184 and k2 = -0.022 from table 60 ,SP : 16
Pb = ( 0.184 - 0.022 x 0.156 ) x 20 x 230 x 450 x 10-3
= 0.181
x
2070
= 373.8
KN
k = ( Puz - Pu ) / ( Puz - Pb )
= ( 1904.7 - 1000 ) / ( 1904.7 - 373.8 )
= 904.7 / 1530.9
= 0.59
Design for
Pu = 1000 KN
Mux = 32 + 0.59 x 39.2 = 55.13 KNm < 80
Muy = 16 + 0.59 x 28 = 32.52 KNm < 40

Take , Mux = 80 KNm


Take , Muy = 40 KNm

For p / fck = 0.162 and Pu / fck b D = 0.48 , the reinforcement being equally distributed ,
the moment capacities can be found out as follows :
About XX
d' / D = 50 / 450 = 0.11
0.15
From chart 45 , SP : 16
Mux1 / fck b D2 = 0.145
Mux1 = 0.145 x 20 x 230 x 4502 x 10-6
= 135.07
KNm
About YY
d' / D = 50 / 230 = 0.22
0.2
From chart 46 , SP : 16
Muy1 / fck b D2 = 0.13
Muy1 = 0.13 x 20 x 450 x 2302 x 10-6
= 61.89
KNm
Check :
Pu / Puz = 1000 / 1904.7 = 0.525
From , IS 456-2000 , Clause 39.6
n = 1.542
check :

Mux n

Mux1

Muy
Muy1

1.542

0.446
0.956

+
1

1.542

80
135.07

0.2 difference
0.075 difference

40
61.89
0.51
.( O.K.)

230

0.34
?

230

Provide 4 - 25 O + 4 - 20 O equally distributed.


Ties : Minimum diameter Otr = 25 / 4
= 6.25 mm
Use 8 mm diameter M.S. ties.
Spacing should not exceed lesser of
( i ) 230 mm
( least lateral dimension )
( ii ) 16 x 25 = 400 mm
( 16 times least longitudinal diameter of bar )
( iii ) 48 x 8 = 384 mm
( 48 times diameter of tie )
Provide 8 mm O M.S. ties @ 230 mm c/c
Note that the distance between corner bars in one face is less than 48 O tr
( 450 - 80 -25 = 345 < 48 x 8 = 384 ). Therefore two sets of Opened ties shall be used .
Note that if this distance would be more than ( 48 x 8 ) mm , closed ties for internal bars would be
sufficient.

IS 456-2000 clause 25

Note :- A column may be considered as short when both slender


lex / D and ley / b 12 OR lex / ixx < 40 where, For Circular column
10 .
( Given )

KNm at bottom.

KNm at bottom.

lex = effective length in respect of the major axis


D = depth in respect of the major axis
ley = effective length in respect of the minor axis
b = width of the member
ixx = radius of gyration in respect of the major axis.
iyy = radius of gyration in respect of the minor axis.

IS 456 : 2000
Table 28 Effective Length of Compression Memb
Degree of End Restraint of
Compression members
Effectively held in position and
restrained against rotation in both
ends

Effectively held in position at both


ends , restrained against rotation
at one ends

Effectively held in position at both


ends , but not restrained against
rotation .
Effectively held in position and restrained
against rotation at one end , and at the
other end restrained against rotation but
not held in position
Effectively held in position and restrained
against rotation at one end , and at the
other partially restrained against rotation
but not held in position
Effectively held in position at one end but
not restrained against rotation and at the
other end restrained against rotation but
not held in position
Effectively held in position and restrained
against rotation at one end but not held
in position nor restrained against rotation
at the other end.

Symbol

Effectively held in position and restrained


against rotation at one end but not held
in position nor restrained against rotation
at the other end.

NOTE - is the unsupported length of compression member.


( i + 1 ) th floor

slab

Beam

( i ) th floor

ly on four sides.

1 of the IS 456-2000

Ac + 0.75 fy Asc

00 - 120000*3/100

116400 x 10-3 + 0.75 x 415 x 3600 x 10-3


1120.5

Effective length of column ( ef ) : It is the distance between the points of zero


column height .

SP : 16 ,Table 60
Slender compression members- Values of P
Rectangular sections : Pb / fck b D = k1 + k2 . p / f
Circular sections : Pb / fck D2 = k1 + k2 . p / fck
Values of k1
Section
0.05

d' / D
0.10
0.15

0.219

0.207

0.196

0.172

0.160

0.149

250
415

0.05
-0.045
0.096

d' / D
0.10
-0.045
0.082

Rectangular ; equal
reinforcement on four
sides

500
250
415
500
250

0.213
0.215
0.424
0.545
0.193

0.173
0.146
0.328
0.425
0.148

Circular

415

0.410

0.323

500

0.543

0.443

Rectangular
Circular
Values of k2
Section

fy N/ mm

Rectangular ; equal
reinforcement on two
opposite sides

values of k .

al moments

IS 456-2000 clause 39.7.1


The additional moments Max and May shall be calculated by the following f

ode IS 456-2000 )

Max = ( Pu D / 2000 ) x ( ex / D )2
May = ( Pu b / 2000 ) x ( ey / b )2
Where ,
Pu = axial load on the member,

nimum eccentricity
tial moment for
mm

> 20 mm

mm

> 20 mm

ex = effective length in respect of the major axis ,


ey = effective length in respect of the minor axis ,
D = depth of the cross - section at right angles to
the major axis , and
b = width of the member
NOTES : 1) A column may be considered braced in a given plane if lateral
stability to the structure as a whole is provided by walls or
bracing or buttressing designed to resist all lateral forces in
that plane. It should otherwise be considered unbraced.
2 ) In the case of a braced column without any transverse loads

occurring in its height, the additional moment shall be added


to an initial moment equal to sum of 0.4 Mu1, and 0.6 Mu2,
where Mu2 is the larger end moment and Mu1 is the smaller
end moment (assumed negative if the column is bent in double
curvature). In no case shall the initial moment be less than
0.4 Mu2 nor the total moment including the initial moment be
less than Mu2. For unbraced columns, the additional moment
shall be added to the end moments.
3 ) Unbraced compression members, at any given level or storey,
subject to lateral load are usually constrained to deflect
equally. In such cases slenderness ratio for each column may
be taken as the average for all columns acting in the same

direction.
IS 456-2000 clause 39.7.1.1
The values given by equation 39.7.1 may be multiplied by the following fa
K = ( Puz - Pu ) / ( Puz - Pb )
1
where ,
Pu = axial load on compression member,
Puz = as defined in 39.6,
Puz = 0.45 fck Ac + 0.75 f
-0.068

Pb = axial load corresponding to the condition


of maximum compressive strain of
0.0035 in concrete and tensile strain of
0.002 in outer most layer of tension steel.

8-25 O

8O@
300 c/c
(two sets )

used .
l bars would be

g)

60 KNm at bottom. ( Given )

d 30 KNm at

re less than

al steel are

-0.1275

4-25 O
+ 4 -20 O
450

l diameter of bar )

e used .
rnal bars would be

8 O @ 230 mm c/c
( three sets )

as short when both slenderness ratios


where, For Circular column lex / D

major axis

minor axis

e major axis.

e minor axis.

th of Compression Members ( Clause E-3 )


Symbol

Theoretical Value
of effective length

Recommended Value
of effective length

0.5

0.65

0.7

0.8

1.0

1.0

1.0

1.2

1.5

2.0

2.0

2.0

2.0

2.0

2.0

f compression member.
slab

Beam

ance between the points of zero moment (contraflexure ) along the

s- Values of Pb
= k1 + k2 . p / fck

k1 + k2 . p / fck
/D
0.20
0.184
0.138
d' / D
0.15
-0.045
0.046

0.20
-0.045
-0.022

0.104
0.061
0.203
0.256
0.077

-0.001
-0.011
0.028
0.040
-0.020

0.201

0.036

0.291

0.056

alculated by the following formulae :

f the major axis ,

f the minor axis ,


at right angles to

en plane if lateral

by walls or

eral forces in
nbraced.

transverse loads

hall be added
nd 0.6 Mu2,
the smaller

is bent in double

be less than
al moment be

ional moment

ven level or storey,


o deflect

ch column may

in the same

ultiplied by the following factor :

5 fck Ac + 0.75 fy Asc

Design of short eccentrically loaded square columns - Biaxial bending.


Size 500 mm x 500 mm
Axial factored load 1500 KN
Factored moment Mux = 90 KNm
( Given )
Muy = 120 Knm
moment due to minimum eccentricity is less than the applied moment.
Material M15 grade concrete
HYSD reinforcement of grade Fe415
Solution :Assume an axial load P'u of 1500 KN and a uniaxial moment M'ux = 90 + 120 = 210 KNm .
P'u / ( fck x b x D ) = 1500 x 103 / ( 15 x 500 x 500 )
= 0.4
M'ux / ( fck x b x D2 ) = 210 x 106 / ( 15 x 500 x 5002 )
= 0.112
d' = 40 + 10 = 50 mm
d' / D = 50 / 500 = 0.1
From chart 32 , SP-16
p / fck = 0.078
p = 0.078 x 15 = 1.17
As = 1.17 x b x D = 1.17 x 500 x 500 / 100
= 2925
mm2
Provide 4-25 mm O + 4-20 mm O = 3220 mm 2 , equally distributed
p = 3220 x 100 / ( 500 x 500 )
p = 1.288
p / fck = 1.288 / 15
= 0.086
The assumed section is now checked.
For p / fck = 0.086 and Pu / ( fck x b x D ) = 0.4 ,
The reinforcement being equally distributed , the moment capacities from
Chart - 44 , SP-16
Mux1 / ( fck x b x D2 ) = Muy1 / ( fck x b x D2 ) = 0.108
Mux1 = Muy1 = 0.108 x 15 x 500 x 5002 x 10-6
= 202.5
KNm
Puz = 0.45 fck Ac + 0.75 fy Asc
Ac = 500 x 500 - 3220
= 246780 mm2
Puz = 0.45 x 15 x 246780 x 10-3 + 0.75 x 415 x 3220 x 10-3
= 1665.77
= 2667.99 KN

1002.23

Pu / Puz = 1500 / 2668


= 0.56
From , IS 456-2000 , Clause 39.6
n = 1.602
check :
n
Mux
Muy
Mux1
90
202.5
0.273

1.602

+
+

Muy1
120
202.5

0.2 difference
0.04 difference

0.34
?

-0.068

1.602

0.433

0.706
1
Ties : Minimum diameter Otr = 25 / 4 = 6.25 mm
Use 8 mm diameter M.S. ties.
Spacing should not exceed lesser of
( i ) 500 mm
( ii ) 16 x 25 = 400 mm
500
( iii ) 48 x 8 = 384 mm
Provide 8 mm O M.S. ties @ 350 mm c/c

500
4-25 O +
4-20 O
8O@
350 c/c

Design of short eccentrically loaded rectangle columns - Biaxial bending.


Size 300 mm x 500 mm
Axial factored load 1500 KN
Factored moment Mux = 60 KNm
( Given )
Muy = 60 KNm
moment due to minimum eccentricity is less than the applied moment.
Material M15 grade concrete
HYSD reinforcement of grade Fe415
Solution :Assume an axial load P'u of 1500 KN and a uniaxial moment M'uy = 300 / 500 ( 60 + 60 ) = 72 KNm .
P'u / ( fck x b x D ) =
=
2
M'u / ( fck x b x D ) =
=
d' = 40 + 10 = 50 mm
d' / D = 50 / 300 = 0.167
From chart 34 , SP-16
p / fck = 0.167
p = 0.167 x 15 = 2.5

1500 x 103 / ( 15 x 300 x 500 )


0.67
72 x 106 / ( 15 x 500 x 3002 )
0.107
..use 0.2

As = 2.5 x b x D = 2.5 x 300 x 500 / 100


= 3750
mm2
Provide 8-25 mm O = 3928 mm2 , equally distributed
p = 3928 x 100 / ( 300 x 500 )
p = 2.62
p / fck = 2.62 / 15
= 0.175
The assumed section is now checked.
About X
d' / D = 50 / 500 = 0.1 , For p / fck = 0.175 and Pu / ( fck x b x D ) = 0.67 ,
Chart - 44 , SP-16
Mux1 / ( fck x b x D2 ) = 0.13
Mux1 = 0.13 x 15 x 300 x 5002 x 10-6
= 146.25 KNm
About Y
d' / D = 50 / 300 = 0.167 say 0.2 , For p / fck = 0.175 and Pu / ( fck x b x D ) = 0.67 ,
Chart - 46 , SP-16
Muy1 / ( fck x b x D2 ) = 0.103
Muy1 = 0.103 x 15 x 500 x 3002 x 10-6
= 69.53
KNm
Pure axial load capacity
Puz = 0.45 fck Ac + 0.75 fy Asc
Ac = 300 x 500 - 3928
= 146072 mm2
Puz = 0.45 x 15 x 146072 x 10-3 + 0.75 x 415 x 3928 x 10-3
= 985.986
+
1222.59
= 2208.58 KN
Pu / Puz = 1500 / 2208.6
= 0.68
From , IS 456-2000 , Clause 39.6
n = 1.802
0.2 difference
check :
0.12 difference
n
n
Mux
Muy
1
M
M
ux1

uy1

1.802
60
146.25
0.2

+
+

1.802
60
69.53
0.77

0.33
?

-0.198

0.97
1
Ties : Minimum diameter Otr = 25 / 4 = 6.25 mm
Use 8 mm diameter M.S. ties.
Spacing should not exceed lesser of
( i ) 300 mm
( ii ) 16 x 25 = 400 mm
300
( iii ) 48 x 8 = 384 mm
Provide 8 mm O M.S. ties @ 300 mm c/c

500

Note that the distance between corner bars in one face is more than 48 O tr
( 500 - 80 -25 = 395 > 48 x 8 = 384 ). Therefore two sets of closed ties shall be used .
Note that if this distance would be less than ( 48 x 8 ) mm , open ties for internal bars
would be sufficient.

0 = 210 KNm .

39.6 Members Subjected to Combined Axial Load


and Biaxial Bending
where ,
Mux

Mux1

Muy
Muy1

Mux1, Muy1 =

n is related to Pu/Puz
where Puz = 0.45 fck Ac + 0.75 fy Asc
For values of Pu / Puz = 0.2 to 0.8, ( 0.2 , 0.4 , 0.6 , 0.8 )the values of

linearly from 1 .0 to 2.0.( 1.0 , 1.33 , 1.67 , 2.0 ) For values less than 0.2,

1 .O; for values greater than 0.8, n is 2.0.

4-25 O +
4-20 O

( least lateral dimension )


( 16 times least longitudinal diameter of bar )
( 48 times diameter of tie )

8O@
350 c/c

0 ( 60 + 60 ) = 72 KNm .

8-25 O

( least lateral dimension )


( 16 times least longitudinal diameter of bar )
( 48 times diameter of tie )

be used .

8O@
300 c/c
(two sets )

Mux , Muy = moments about x and y axes


due to design loads,
Mux1, Muy1 = maximum uniaxial moment
capacity for an axial load of Pu,
bending about x and y axes
respectively, and

.6 , 0.8 )the values of n vary

) For values less than 0.2, n is

Design of short eccentrically loaded columns - uniaxial bending.


Size 300 mm x 600 mm
Axial factored load 600 KN
Factored moment 300 KNm
( Given )
moment due to minimum eccentricity is less than the applied moment.
Material M15 grade concrete
HYSD reinforcement of grade Fe415
Solution :Using 25 mm diameter bars with 40 mm clear cover
d' = 40 + 12.5 = 52.5 mm
d' / D = 52.5 / 600 = 0.088 ,say 0.1
Pu / ( fck x b x D ) = 600 x 103 / ( 15 x 300 x 600 )
= 0.222
2
Mu / ( fck x b x D ) = 300 x 106 / ( 15 x 300 x 6002 )
= 0.185
From chart 32 , SP : 16
p / fck = 0.1
p = 0.1 x 15 = 1.5
As = 1.5 x b x D = 1.5 x 300 x 600 / 100
= 2700
mm2
Provide 22 mm O ( 2700 / 380 = 8 no.) = 3041 mm 2
As the distance between two opposite corner bars is more than 300 mm ,
provide 2- 12 mm O at centre of long sides of the column, which may be tied by open ties.
Provide 8- 25 O + 2 - 12 O = 4151 mm2
600
Ties : Minimum diameter Otr = 25 / 4 = 6.25 mm
Use 8 mm diameter M.S. ties.
Spacing should not exceed lesser of
( i ) 300 mm
300
( ii ) 16 x 25 = 400 mm
( iii ) 48 x 8 = 384 mm
Provide 8 mm O M.S. ties @ 300 mm c/c ( two sets )
Note that the distance between corner bars in one face is more than 48 O tr
( 600 - 80 -25 = 495 > 48 x 8 = 384 ). Therefore two sets of closed ties shall be used .
Note that if this distance would be less than ( 48 x 8 ) mm , open ties for internal bars
would be sufficient.

d by open ties.

8-25 O
+ 2-12 O
8 mm O @
300 mm c/c
( two sets )

be used .

( least lateral dimension )


( 16 times least longitudinal diameter of bar )
( 48 times diameter of tie )

Design of short circular column


Working load = 1200 KN
Assume emin < 0.05 D
( a ) lateral ties &
( b ) helical reinforcement
Material M20 grade concrete
HYSD steel Fe415
For Lateral reinforcement mild steel Fe250
Solution : Factored load = 1.5 x 1200
= 1800
KN
( a ) lateral ties : Pu = 0.4 fck Ac + 0.67 fy Asc
Assume 0.8 % minimum steel.
Asc = 0.008 Ag
Then ,

Given

Ac = Ag - Asc
= 0.992 Ag
Substituting , we have
0.4 x 20 x 0.992 Ag + 0.67 x 415 x 0.008 Ag
1800 x 103 =
1800 x 103 =

7.936

Ag +

1800 x 103 =

10.1604

Ag

2.2244

Ag

Ag = 177165 mm2
If D is the diameter of the column
( / 4 ) x D2 = 177165
D = 225688
D = 475 mm
Use 475 mm diameter column.
Asc = 0.008 x 177165
= 1417
mm2
Minimum 6 bars shall be used.
Provide 16 mm diameter bars ( 1417 / 201 ) = 8 no.
Asc = 8 x 201
= 1608
mm2
Use 6 mm O lateral ties , Spacing shall be lesser of
( i ) 475 mm
( least lateral dimension )
( ii ) 16 x 16 = 256 mm
( 16 times least longitudinal diameter of bar )
( iii ) 48 x 6 = 288 mm
( 48 times diameter of tie )
Provide 6 mm O lateral ties @ 250 mm c/c .
( b ) Helical reinforcement : The column with helical reinforcement can support 1.05 times the load of a similar
member with lateral ties. Therefore
Pu = 1.05 [ 0.4 fck Ac + 0.67 fy Asc ]

1800 x 103 = 1.05 [0.4 x 20 x 0.992 Ag + 0.67 x 415 x 0.008 Ag ]


Ag +
1800 x 103 = 8.3328
2.33562 Ag
1800 x 103 = 10.66842 Ag
Ag = 168729 mm2
If D is the diameter of the column
( / 4 ) x D2 = 168729
D = 214941
D = 463 mm
Use 450 mm diameter column.
Ag = ( / 4 ) x 4502
Then ,
= 158962.5 mm2
1800 x 103 = 1.05 [0.4 x 20 x (158963 - Asc ) + 0.67 x 415 x Asc ]
= 1335289

8.4

Asc

Asc
=
283.55
Asc = 1638.903 mm2
provide 20 mm diameter bars ( 1638.9 / 314 =) 6 No.
Asc = 6 x 314
464711

= 1884
mm2
Assume 8 mm O M.S. bars for helix at 40 mm clear cover .
Dc = 450 - 40 - 40
= 370
mm
asp = ( / 4 ) x 82
= 50
mm2
Minimum s = 0.36 ( (Ag / Acr) - 1 ) fck / fy
= 0.36 ( ( 4502 / 3702 ) - 1 ) x 20 / 250
= 0.36 x 0.4792 x 20 / 250
= 0.013801
s = 4 x asp / p x Dc
Now
0.0138 = 4 x 50 / p x 370
p = 39.17
mm
.. ( 1 )
As per IS 456-2000 clause 26.5.3.2 ( d )
The pitch < 75 mm
< Dc / 6 ( = 370 / 6 = 61.67 mm )
> 25 mm
> 3 x dia of helix bar = 3 x 8 = 24 mm ( 2 )
From ( 1 ) and ( 2 ) , provide 8 mm O helix @ 35 mm pitch.

291.953 Asc

al diameter of bar )

6 - 20 O

450

6 - 20 O

35
8 mm O @
35 mm c/c

Design of short column


Factored load = 1500 KN
Assume emin < 0.05 D

Given

Material M15 grade concrete


mild steel Fe250
Solution : Here , emin < 0.05 D , But emin = 20 mm

Therefore , size of column shall be minimum ( 20 / 0.05 = 400 )

400 mm x 400 mm

Assume 0.8 % minimum steel.


Asc = 0.008 Ag
Then ,
Ac = Ag - Asc
= 0.992 Ag
Pu = 0.4 fck Ac + 0.67 fy Asc
= 0.4 x 15 x 0.992 Ag + 0.67 x 250 x 0.008 Ag
Ag
5.952 Ag +
1.34
1500 x 103 =
1500 x 103 =
7.292 Ag
Ag =

205705 mm2

If the column is to be a square , the side of column = 205705 = 453 mm


Adopt 450 mm x 450 mm size column .
Then ,
1500 x 103 = 0.4 x 15 x ( 450 x 450 - Asc )+ 0.67 x 250 x Asc
Asc
+
167.5
1500 x 103 = 1215000 - 6 Asc
Asc
285000
=
161.5
Asc = 1765
mm2
Provide 20 mm diameter bars = ( 1765 / 314 ) = 6 No.
giving , Asc = 6 x 314 = 1884 mm2
Note that the distance between the bars exceeds 300 mm on two parallel sides .
the arrangement of reinforcement should be changed.
Provide then 4 no. 20 mm diameter bars plus 4 no. 16 mm diameter bars giving
Asc = 4 x 314
+
4 x 201
= 1256

804

= 2060
mm2
Lateral ties :
Use 6 mm O lateral ties.
Spacing should be lesser of :
( i ) 450 mm
( least lateral dimension )
( ii ) 16 x 16 = 256 mm
( 16 times least longitudinal diameter of bar )
(iii) 48 x 6 = 288 mm .
( 48 times diameter of tie )
Provide 6 mm O ties about 250 mm c/c .

Note that the distance between corner bars in one face is more than 48 O tr
( 450 - 80 -20 = 350 > 48 x 6 = 288 ). Therefore two sets of closed ties shall be used .
Note that if this distance would be less than ( 48 x 6 ) mm , open ties for internal bars
would be sufficient.
450

450

6 - 20 O

4 - 20 O
+ 4 - 16 O
450

450

6 mm O @ 250 mm c/c
(double ties )

wrong arrangement

correct arrangement

IS 456-2000 clause 25
Note :- A column may be considered as short when both slenderness ratios
lex / D and ley / b 12 OR lex / ixx < 40 where,
lex = effective length in respect of the major axis
D = depth in respect of the major axis
ley = effective length in respect of the minor axis
x 400 mm

b = width of the member


ixx = radius of gyration in respect of the major axis.
iyy = radius of gyration in respect of the minor axis.
All columns shall be designed for minimum eccentricity equal to the
unsupported length of column / 500 plus lateral dimension / 30 , subject to
a minimum of 20 mm. For bi-axial bending this eccentricity exceeds the
minimum about one axis at a time.
IS 456-2000 clause 39.3
When emin 0.05 D , the column shall be designed by the following
equation:
Pu = 0.4 fck Ac + 0.67 fy Asc
Ac = Area of concrete ,
Asc = Area of longitudinal reinforcement for columns.
If emin > 0.05 D , the column shall be designed for moment also.
IS 456-2000 clause 26.5.3.1
The cross-sectional area of longitudinal reinforcement , shall be not less
than 0.8 % nor more than 6 % of the gross cross-sectional area of the
column.
NOTE - The use of 6 percent reinforcement may involve
practical difficulties in placing and compacting of concrete;
hence lower percentage is recommended. Where bars from
the columns below have to be lapped with those in the
column under consideration, the percentage of steel shall
usually not exceed 4 percent
26.5.3 columns
26.5.3.1 Longitudinal reinforcement
a) The cross-sectional area of longitudinal
reinforcement, shall be not less than 0.8 percent
nor more than 6 percent of the gross crosssectional
area of the column.
NOTE - The use of 6 percent reinforcement may involve
practical difficulties in placing and compacting of concrete;
hence lower percentage is recommended. Where bars from

be used .

4 - 20 O
+ 4 - 16 O

6 mm O @ 250 mm c/c
(double ties )

the columns below have to be lapped with those in the


column under consideration, the percentage of steel shall
usually not exceed 4 percent.
b) In any column that has a larger cross-sectional
area than that required to support the load,
the minimum percentage of steel shall be
based upon the area of concrete required to
resist the direct stress and not upon the actual
area.
c)The minimum number of longitudinal bars
provided in a column shall be four in rectangular
columns and six in circular columns.
d )The bars shall not be less than 12 mm in
diameter.
e) A reinforced concrete column having helical
reinforcement shall have at least six bars of
longitudinal reinforcement within the helical
reinforcement.
f )In a helically reinforced column, the longitudinal
bars shall be in contact with the helical
reinforcement and equidistant around its inner
circumference.
g )Spacing of longitudinal bars measured along
the periphery of the column shall not exceed
300 mm.
h )In case of pedestals in which the longitudinal
reinforcement is not taken in account in strength
calculations, nominal longitudinal reinforcement
not less than 0.15 percent of the cross-sectional
area shall be provided.

slenderness ratios

equal to the
on / 30 , subject to
ity exceeds the

e following

shall be not less


nal area of the

Design of dog-legged staircase


230
UP
Floor

9 10

A
20
v
vv
300
vv

900
B

19

900

15

900

230
230

> 200 mm
Tread < 230 mm

Rise of step = 160 mm

1950

900

11

2250

150

Rise

175 mm to 200 mm

Tread = 250 mm
250 mm to 280 mm
Nosing is not provided
( given )
Material M15 Grade concrete
mild steel reinforcement Fe250
Solution : Assume 150 mm thick waist slab. Both the landings can span on walls.
Landing A or B
Self-load 0.15 x 25 = 3.75
KN / m2
Floor finish = 1.00
KN / m2
Live load ( residence ) = 3.00
KN / m2
Total 7.75
KN / m2
PU = 1.5 x 7.75 =
11.63 KN / m2
Span =
1950 + 150 = 2100
i.e. 2.1 m
Consider 1 m length of slab
M = w x l2 / 8
= 11.63 x 2.12 / 8
= 6.41
KNm
Reinforcement will be in second layer, Assuming 12 mm O bars
d = 150 - 15 (cover ) -12 - 6
= 117
mm
Mu / b x d2 = 6.41 x 106 / 1000 x 1172
= 0.468
Pt = 50 1-1-(4.6 / fck) x (Mu / b x d2)
fy / fck
= 50 1-1-(4.6 / 15) x (0.468)

250 / 15
= 50 [(1-0.93) x 15 / 250 ]
= 0.224%
Ast = 0.224 x 1000 x 117 / 100
= 262
mm2
According to IS 456-200 clause 26.5.2.1 Minimum steel 0.15 % for Fe250 and 0.12 %
for Fe415
Minimum steel = ( 0.15 / 100 ) x 1000 x 150
= 225
mm2
Provide 10 mm O bar
spacing of bar = Area of one bar x 1000 / required area in m 2 /m
= 78.50 x 1000 /262
= 300
mm
Provide 10 mm O bar @ 280 mm c/c = 280
Maximum spacing = 3 x d = 3 x 117
= 351

mm2 .
mm

.( O.K.)

Check for shear : Vu = w x l / 2


= 11.63 x 2.1 / 2
= 12.21
KN
Shear stress = Vu / b x d
= 12.21 x 103 / 1000 x 117
= 0.104

N / mm2

for Pt = 0.224 c = 0.28

< ( ) N / mm ( too small )


< 0.28 ( from table 7-1 )
2

.( O.K.)
Check for development length : Assuming L0 = 12 O
(mild steel )
Pt = 100 x As / b x d
= 100 x 280 / 1000 x 117
= 0.239
From equation
Pt = 50 1-1-(4.6 / fck) x (Mu / b x d2)
fy / fck
Mu1 / b x d2 = 0.499
Mu1 = 0.499 x 1000 x 1172 x 10-6
= 6.83
KNm
Vu = 12.21
KN
1.3 x ( Mu1 / Vu ) + L0 Ld

1.3 x ( 6.83 x 106 / 12.21 x 103 ) + 12 O 55 O


727.191 + 12 O 55 O
which gives
Check for deflection : -

43 O 727.19
O 16.91

mm

.( O.K.)

Basic ( span / d ) ratio = 20


Pt = 100 x Ast / b x d = 100 x 280 / 1000 x 117
= 0.239
IS 456-2000 clause 23.2.1 fig-4 ,for tension reinforcement
modification factor = 2
( span / d ) ratio permissible = 2 x 20
= 40
Actual (span / d ) ratio = 2100 / 117
=
17.95

<

40

.( O.K.)

Design of flight : Span and loading on both the flights are the same. Therefore same design will be
adopted.
Loads :
Inclined length of waist slab for one step = 2502 + 1602
= 296.8
mm
Assuming 150 mm thick waist slab
self-load in plan = ( 296.8 / 250 ) x 0.15 x 25 = 4.45
Floor finish length for one step = 160 + 250
= 410
mm

KN / m2

floor finish = ( 410 / 250 ) x 1 = 1.64


Weight of step ( ( 0 + 160 ) / 2000 ) x 25 =

KN / m2
250
2.00
KN / m2
160
296.8
2
3.00
KN / m
4.45 + 1.64 + 2.00 + 3.00
KN / m2
11.09

Live load =
Total =
=
Pu = 1.5 x 11.09 = 16.64

KN / m2

IS 456-2000 , clause 33.2


Landing is spanning in transverse direction. The span of stair according to discussion
made in art 13-1 and loading for 1 m width of stair is shown in fig below.
16.64 KN / m
11.63 KN / m

11.63 KN / m

RA = 24.83 KN

RB = 24.83 KN
525

2250

525

525 = 900 / 2 +75

3300

RA = RB = 0.525 x 11.63 + 16.64 x 2.25 / 2


=
6.11
+
18.72
= 24.83
KN
Mu = 24.83 x 3.3/2 - 11.63 x (3.3/2)2 / 2 - ( ( 16.64 - 11.63 ) x ( 2.25/2 )2 ) / 2
= 40.97 - 15.83 - 3.17
= 21.97
KNm
From Table 6-3
Q = 2.22
drequired =
=
=
dprovided =
=
Mu / b x d2 =
=

M / Q x b
21.97 x 10
99.48

/ 2.22 x 1000

mm,

150 - 15 - 6
129
mm,

.( O.K.)

21.97 x 10 6 / 1000 x (129)2


1.32

Pt = 50 1-1-(4.6 / fck) x (Mu / b x d2)


fy / fck
= 50 1-1-(4.6 / 15) x (1.32)
250 / 15
= 50 [(1-0.77) x 15 / 250 ]
= 0.687%
Ast = 0.687 x 1000 x 129 / 100
= 886
mm2
Provide 12 mm O bar
spacing of bar = Area of one bar x 1000 / required area in m 2 / m
= 113.04 x 1000 / 886
= 127.585
mm
Provide 12 mm O bar@125 mm c/c = 904
mm2.
Minimum steel is 0.15 % for mild steel and 0.12 % for HYSD Fe415 reinforcement
Distribution steel = ( 0.15 / 100 ) x 1000 x 150
= 225
mm2
Provide 8 mm O bar
spacing of bar = Area of one bar x 1000 / required area in m 2 /m
= 50.24 x 1000 /225
= 223
mm
Provide 8 mm O bar @ 220 mm c/c = 228
mm2 .

Check for shear : Vu = 24.83

KN

Shear stress = Vu / b x d
= 24.83 x 103 / 1000 x 129
= 0.192

N / mm2

< (

) N / mm2

( too small )

100 x As / b x d = 100 x 904 / 1000 x 129


= 0.70
from table 7-1
for Pt = 0.7
c = 0.524 N / mm2
IS 456-2000 clause 40.2.1.1
k = 1.3
for 150 mm slab depth
Design shear strength = 1.3 x 0.524
= 0.681
N / mm2 .( O.K.)
Check for development length : Assuming L0 = 12 O
(mild steel )
Pt = 100 x As / b x d
= 100 x 904 / 1000 x 129
= 0.70
From equation
Pt = 50 1-1-(4.6 / fck) x (Mu / b x d2)
fy / fck
we get ,

Mu1 / b x d2 = 1.344
Mu1 = 1.344 x 1000 x 1292 x 10-6
= 22.37
KNm
Vu = 24.83
KN

Development length of bars Ld =

O s / 4 x bd

= O x 0.87 x 250 / 4 x 1

=54.4 O

1.3 x ( Mu1 / Vu ) + L0 Ld
1.3 x ( 22.37 x 106 / 24.83 x 103 ) + 12 O 54.4 O
1171.2 + 12 O 54.4 O
42.4 O 1171.2
which gives
O 27.62
mm
.( O.K.)
From Crossing of bars, the bars must extend upto 54.4 x 12 = 653
say 700 mm.
Check for deflection : Basic ( span / d ) ratio = 20
Pt = 100 x Ast / b x d = 100 x 904 / 1000 x 129
= 0.70
IS 456-2000 clause 23.2.1 fig-4 ,for tension reinforcement
modification factor = 1.62
( span / d ) ratio permissible = 1.62 x 20

= 32.4
Actual (span / d ) ratio = 3300 / 129
= 25.58
<
32.4
.( O.K.)
The depth could be reduced
Check for cracking : IS 456-2000 , clause 26.3.3
Main bars : maximum spacing permitted = 3 x effective depth of slab or 300 mm whichever is sm
= 3 x 129 = 387
mm
or 300 mm
spacing provided = 125
mm
< 300 mm
.( O.K.)
Distribution bars : maximum spacing permitted = 5 x effective depth of slab or 450 mm which
= 5 x 129 = 645
mm
spacing provided = 220
mm
< 450 mm
.( O.K.)

to 200 mm
to 280 mm

check for shear


IS 456-2000 , Table 19
Table 7-1
Design shear strength of concrete , C, N / mm2
Pt = 100 x As
bxd
0.15
0.25

Concrete grade
M15
0.28
0.35

M20
0.28
0.36

M25
0.29
0.36

M30
0.29
0.37

M35
0.29
0.37

0.50
0.75

0.46
0.54

0.48
0.56

0.49
0.57

0.50
0.59

0.50
0.59

1.00

0.60

0.62

0.64

0.66

0.67

1.25
1.50
1.75

0.64
0.68
0.71

0.67
0.72
0.75

0.70
0.74
0.78

0.71
0.76
0.80

0.73
0.78
0.82

2.00

0.71

0.79

0.82

0.84

0.86

2.25
2.50
2.75
3.00

0.71
0.71
0.71
0.71

0.81
0.82
0.82
0.82

0.85
0.88
0.90
0.92

0.88
0.91
0.94
0.96

0.90
0.93
0.96
0.99

The above given table is based on the following formula


Design shear strength c = 0.85 0.8 x fck ( 1 + 5 x - 1 )
6x
= 0.8 x fck / 6.89 Pt , but not less than 1.0
IS 456-2000 , Table 20
Table 7-2

Maximum shear stress , C, N / mm2


M30

M35

c )max N / mm
2.5
2.8
3.1
3.5
check for development length
IS 456-2000 clause 26.2.1
Development length of bars Ld = O s / 4 x bd

3.7

Concrete grade
(

M15

M20

M25

IS 456-200 clause 26.2.1.1


Table 7-5
Design bond stress (bd ) for plain bars in tension
Concrete grade
(

M15

M20

M25

M30

M35

bd N / mm2

1.0
1.2
1.4
1.5
1.7
Note-1 : bd shall be increased by 25 % for bars in compression
Note-2 : In case of deformed bars confirming to IS : 1786-1985 , the
value of bd shall be increased by 60 %.
For mild steel Fe250
For Fe415
IS 456-2000 clause 26.2.3.3
Ld M1 / V + L0

s = 0.87 x fy
s = 0.67 x fy

L0 = effective depth of the members or 12 O , whichever is greater


if ends of the reinforcement are confined by a compressive reaction M1 /
V increased by 30 % ( 1.3 M1 / V )
Table 7-6
Development length for single mild steel bars
Tension bars Compression bars
fy N / mm2
M15
M20
M15
M20
250
55 O
26 O
44 O
37 O
415
56 O
47 O
45 O
38 O
500
69 O
58 O
54 O
46 O
check for deflection
Basic values of span to effective depth ratios for spans upto 10 m :
cantilever
7
simply supported
20
continuous
26
For spans above 10 m, the values in (a) may be multiplied by
10 / span in metres , except for cantilever in which case deflection
calculations should be made.
check for cracking
(a)The horizontal distance between parallel main

reinforcement bars ( spacing )shall not be more than t


times the effective depth of solid slab or
300 mm whichever is smaller.
(b)The horizontal distance between parallel
reinforcement bars ( spacing ) provided against
shrinkage and temperature (distribution bar) shall not be more
than five times the effective depth of a solid
slab or 450 mm whichever is smaller.
IS 456-2000 Clause 40.2.1.1
300 or
Overall depth of
slab , mm
more
275
k
1.00
1.05

250
1.10

225
1.15

sketch :
The reinforcements for both the flights are shown in fig.
700
700
10 O @ 280 c/c
( Landing )

v v
v v
v

200
1.20

10 O @ 280 c/c
( Landing )

vv v
v v

12 O @ 125 c/c
150 mm thick waist slab

8 O @ 220 c/c

v
v v

12 O @ 125 c/c

Flight-B

vv
v
v
v
v
v
v

8 O @ 220 c/c

v
v

12 O @ 125 c/c
150 mm thick waist slab

10 O @ 280 c/c
( Landing )

Flight-A

or 300 mm whichever is small


or 300 mm
i.e. 300 mm
.( O.K.)
h of slab or 450 mm whichever is small
or 450 mm i.e. 450 mm
.( O.K.)

N / mm2
M40
0.30
0.38
0.51
0.60
0.68
0.74
0.79
0.84
0.88
0.92
0.95
0.98
1.01
1+5x-1)

ss than 1.0

m2
M40
4.0

n tension
M40
1.9

mpression
786-1985 , the

er is greater

ssive reaction M1 /

s upto 10 m :

plied by
se deflection

&

For checking development length , l0


may be assumed as 8 O for HYSD
bars ( usually end anchorage is not
provided ) and 12 O for mild steel ( U
hook is provided usually whose
anchorage length is 16 O.

175
1.25

150 or
less
1.30

vv

150

vv

v v v v
v v v

vv
vv
10 O @ 280 c/c
( Landing )
12 O @ 125 c/c
8 O @ 220 c/c

Вам также может понравиться